Download as docx, pdf, or txt
Download as docx, pdf, or txt
You are on page 1of 111

REVIEWER – 1

AUGUST 20, 2014


d. Bronchial tapping
My set 2 had computations only. Sa sangkatutak na
NORMAL values na minimorize ko,iIsa lang lumabas 8. Manifestations of MgSO4 toxicity?
KKLK! Abnormal Value pa sa mai Allergic reaction > < a. Hypertension
Just want to share the blessings that I also received b. Absence of knee jerk
#STUDYHARD #PRAYHARDER
#expatmember 9. Patient with pulmonary heart problem showing
signs of short breath with pedal edema and bluish
STUDY :#SEPTIC SHOCK discoloration on extremities? ( just the thought of the
#HYPOVOLE MICSHOCK actual question)
#ANAPHYLACTICSHOCK a. Cor pulmonale
b. Pulmonary edema
1.Patient called the clinic complaining of UTI, what c. Left sided chf
factors make the nurse to recommend an d. pneumonia
appointment for her.
a. suprapubic tenderness 10. Stoma on colostomy with blanch appearance
b. Pregnancy indicates?
c. Vaginal bleeding a. irritation on gastric secretions
d. all of the above b. infection
c. poor oxygenation
2. Emphysema with hypercarbnia: appropriate oxygen
treatment? 11. Biphasic defibrillator. Initial amount joules?
a.3-5 Ipm via nasal prong a. 50
b.30% via venture mask b. 80
c. 2-4 Ipm via nasal prong c. 200
d.40% via venture mask d. 360

3. What is the correct nursing documentation 12. When do u perform BSE to a pre-menopausal
a. orange mass like upon palpitation woman:
b. right lower quadrant pain in abdomen a. at the MIDPOINT of menstrual CYCLE
c. pt says he has a headache b. at the FIRST DAY of menstraul PERIOD
d. pt says he feels like nauseous c. AFTER the LAST menstrual PERIOD
d. BEFORE the menstrual PERIOD
4. What is Transient Ischemic Attack? (question was
not exactly like this, it's too long, I can't remember 13. Patient with cataract manifests?
how a. blurry of vision
exactly it was constructed but it was in a situation b. presence of floaters
question type)
a.Permanent neurological deficits 14. Patient with Xanthelasma. Indicates? (edited na
b.Temporary deficits that will naturally mahaba to sa exam ko)
c. Temporary deficit with motor problems a. high bilirubin
b. high cholesterol
5. What is ascites? (question was not exactly like
this, it's too long, I can't remember how exactly it 15. Patient with cardiac problem of right back up flow
was constructed but it was in a situation question of blood: manifest?.
type but this is the thought) a.dyspnea
a. Fluid Accumulation on Peritoneal Cavity899 b.crackles
b. Fluid Accumulation related to edema c. hemoptysis
d.pedal edema
6. Blood transfusion: fluid to use before and after
transfusion? 16. A patient is diagnosed with cystic fibrosis.
a. 0.9 % Normal saline Which of the following is contraindicated? (same)
b. DSLR a.chest physiotherapy
c. DSW ater b.posturaI drainage
d. All of the above c. steam inhalation
d.deep breathing exercise
7. Child with cystic fibrosis without complication: what
to do first?. 17. Patient for OR. Morphine and Atropine is given
a. Cough for?.
b. Deep breathing a.to relax muscle
c. Postural Drainage b.induce amnesia
c. decrease oral secretions
29. Patient on enema complain of abdominal
18. Child with croup for discharge. What should cramps?.
be included on parents teaching? a.stop and continue when subsides
a.follow prescribed antibiotics b put the solution on higher
b.have cold mist tent while child is sleeping
30. Patient with risk of for impaired skin integrity due
19. Chemopatient with vomiting. Nursing to immobility. Nursing objective?
intervention? a.provide mouth care and emotional a. monitor intake and output
support b.give a good tasting snack to improve b. absence of redness on skin
appetite
31. Old patient with Diabetes Insipidus. What to
20. Purpose of Penicillin in Rheumatic expect?
fever? a.prevent cardiomyopathy a. hyponatremia
b.kill microorganism and cure the disease b. high urine osmolality
c. prevent reinfection and recurrent of fever
32. Digoxin toxicity. ECG shows?
21. Patient with chest tube. Tidaling in water-seal a. Elevated ST segment
bottle indicates? ( edited) b. Inverted T wave
a.Document as normal finding c. prolonged PR interval
b.Contact physician
c. Check for leakage in the system 33. What is first degree heart block?
Answer: Delay Of Impulse In AV
22. pt complains of as a sign of NODE
hypothyroidism: ( can't recall all the choices but
the others are for hyper) 34. Patient on anticoagulant. Patient needs further
a. I feel hot all the time teaching if?
b. I feel tired a. use electric razor
c. I have a diarrhea b. check feet for bruising
c. states that dark color urine is normal
23. Patient post leg operation. Best way to prevent
thrombophlebitis? 35. Most common sign of fluid volume deficit?.
a.elevate leg with two pillows a. thirst
b.apply leg stocking during night b. dry skin
c.massage frequently c. dark urine
d. anticoagulant
36. Diagnostic test to help diagnose Thalassemia?
24. Patient on Coumadin. What to check? a. PTT
a. APTT b. HGB Electrophoresis
b. CBC c. CBC
c. PT/INR
37. Hemophiliac patient. Normal?
25. Patient on digoxin and loop diuretic. What to a. PT combustion
check to prevent complication? b. PT
a. potassium level c. PTT
b. intake and output
38. Patient on narcotics. What is alarming? (edited)
26. Reason for false High BP reading? (edited ques) a. Lethargy and drowsiness
a. Wrong position of arm or leg b. slight fatigue
b. Inaudible sound
c. Cuff too loose 39. It is done to assess infant's weJI being in
relationship with its own movement inside womb.
27. Position of child with posterior upper lobe a. Contraction stress test
secretions? ( situation ito, 2 questions this is just one b. Non-stress test
of the ques, sorry can't remember the other one) c. Chronic villi sampling
a. leaning forward with pillow
b. supine with legs elevated 40. What is the indication of giving Rhogam to
mother after delivery?
28. Child with pyloric stenosis. What would be a. ABO incompatibility
the manifestations? b. Spontaneous Abortion
a.diarrhea
b.regurgitation 41. Patient with Pilonidal sinus infection. You should
c. projectile vomiting check
a. Base of spine
b. Anal area
51. In the first 24 hours of life, appearance of yellowish
c. Groin area
skin discoloration noted, what is the cause?
d. Lower extremities
a. pathologic jaundice
42. Raynaud's disease. What to avoid? (edited
b. Physiologic jaundice
question mahaba to kaya magbasa)
c. immature liver
Answer: Cold Shower / Exposure
52. DM type 2 occurs mostly?
43. Patient on CTT for X-ray. The nurse will? (edited)
a. Early adulthood
a. clamp the tube b. Middle adulthood
b. maintain the system below chest level
c. late adulthood
44. Sterile urine sample collection?
53. Post-op patient with skin traction is complaining
a. catheterize patient
of tingling & burning in his leg, what the appropriate
b. Void directly to container
action of the nurse?
Massage the leg
45. You noticed that the other nurse made a wrong
Elevate the led
entry on nurse's notes of other nurse what you should Call the doctor
do?
a. You erase the entry
54. Which patient will you consider on URGENT?
b. You let the other nurse erase the entry
a. Closed fracture with superficial lacerations
c. You or the other nurse must draw line, put error
b. Open fracture with distal pulses
then you both sign.
c. Pneumothorax
d. Let the other nurse must draw line, put error
d.Multiple injuries and shock
then sign
55. Temperature of ear irrigation?
46. Nurse have accidentally given medication to a
a. 38.6 C
wrong patient ( mahahaba choices yan Ing maalala
b. 36.8 C
ko, wala talaga akong SATA)
a. Document on patient's chart the medication
56. Best position for newborn while sleeping?
given
a. Supine
b. Write on nurse's note that an incident report is
b. Prone
made
c. left side lying
c. Write on nurse's note that the incident was
made and that the med error was documented on d. right side lying
the med chart
57. PICC, before initiating medication for the first
lime, what the nurse do?
47. A healthy Adult would drink an average of
a. Withdraw blood before giving the medication
in a day and gets 800ml fluid ingested from food
a. 1500mI
b. X-ray to confirm placement
b. 2L
58. Patient is allergic to Penicillin, what medication to
c. 3L
avoid?
d. 5L
a. Ceph alosporin
48. Transmission of Thypoid fever? b. Quinolones
a. Water and food c. Tetracycline
b. Blood contact
59. Which vaccine if given wi[hin 4 weeks
49. Patient with pacemaker you instruct him to?
with tuberculin test, will affect (false results/
a. Avoid Microwave once the pacemaker is operating
weaken effectiveness)
tuberculin test:
(yan pala ung choice from the files kasi during
operation sabi)
a. MMR
b. Always to wear loose cloth around pacemaker
b. DPT
c. Report to physician prolong episodes of c. OPI
hiccups
60. Patient called the clinic complaining of UTI, what
50. In severe burns, you expect the following to factors make the nurse to recommend an
be elevated: appointment for her.
a. Glucose a. suprapubic tenderness
b.Sodium b. Pregnancy
c. Potassium c. Vaginal bleeding
d. all of the above d. All of the above
61. Myringotomy 2 weeks post op patient teaching: approaching?
what to avoid? (lurky ako sa ques na ito, SATA nlng a. talk loudly in the ear
sana Lord kako, wala pang all of the above struggle) b. talk before entering and leaving the room
a. air travel only
b. showering and drinking with straw 72. Which cases indicates a shock with defibrillator?
c. coughing, air travel drinking with straw a.VT with pulse
d. shower, vigorous coughing and drinking with b. VT without pulse
straw c. Supraventricular fibrillation
d. all of the above
62. With diuretics administration, the nurse must be
aware of?
73. Oral hypoglycemics (glypiride) when to
a. Paresthesia and irritability
administer? ( iba choices ko)
b.Muscle weakness and increase
a. After meals at each night
bp
b. With meals
c. Weak pulse
c. with the 1st meal
d. Three times a day everyday
63. A schizophrenic patient is telling the nurse that
everyone wants to hurt him. Which type of delusion
addressing this? 74. Folic acid during pregnancy prevents?
a. Persecution Answer: Neural tube defects
b. Grandeur
75. Tetracycline for 12 year old must not be
64. The most common cause of Melena is ? given since?
a. Colorectal cancer Answer: Staining of teeth
b. Anal fissure
c. Peptic Ulcer Disease 76. 2 weeks post-surgery, a patient return to the clinic
d. Hemorrhoid for his follow up check-up, during assessment of the
wound, the nurse knows that a characteristic of a
65. Patient has amputation of her left leg: having pain wound favorable for bacterial growth is?
in her lower left leg , nurse should? a. moist, beefy tissue
a.explain this called phantom, common after the b. eschar tissue
surgery and its normal c. non-approximate wound edges
b.Give analgesic
c. c call the doctor 77. Patient visited at clinic BP-180/120? What is the
d.Tell pt its normal and it will be subside
priority expected outcome with the visit?
a. Contact Physician
66. In acute bacterial meningitis , the CSF
b. Immediate Hospitalization
investigation will be? c. prescription of nifedipine
b. Dec glucose level
c. Dec protein level
78. All except one are true regarding the overdose of
Pethidine?
68. A pregnant woman is diagnosed with mild
a. Hypotension
pregnancy-induced hypertension. What diet would the
b. Respiratory depression
nurse recommend? c. Pinpoint pupils
a. strict low sodium diet d. Tinnitus
b. increase protein intake
c. maintain a well-balanced diet
79. What is the purpose of debridement of wound
a. to promote tissue granulation
69. What do you need to do before administering b. to remove the bacteria and promote wound
medication? healing
a. Check expiration date
b. check availability
c. check medical order for accuracy 80. Patient came in the ER with heat stroke and the
d. Check dosage and route nurse apply ice pack to groin neck and armpit. Good
outcome if the patient is:
a. Alert and conscious
70. Old patient with visual problem wants to go to CR. b. I feel cold now
Appropriate nursing action?
a. let the patient hold on your elbow and first walk
forward 81. Pre-operative checklist when to do?
b. clear the way to CR and walk together with the c. upon admission
patient d. b before transfering to or
e. pre- admission
71. Patient who is blind. Appropriate way of
82. Post amputation pt, all of the ff are complications 90. What are the Symptoms of alcohol withdrawal
EXCEPT? syndrome?
A. Neuroma
B. Joint enlargement Diaphoresis, Tremors, nausea
C. Flexion contractures
91. Duodenal Peptic ulcer symptoms? Burning Pain
D. Phantom pain
relieved by food

83. Pt has 2 chest tube bottles on the right side, but


the nurse observed that the trachea is pointing 92. Patient with Angina Pectoris came to the ER with
towards the upper left chest, what is the FIRST thing headache, dizziness, palpitations. What to suspect?
Overdose of Nitroglycerine tablet
the nurse should do?
a. Inform the Emergency response team and the
thoracic surgeon 93. Care of meningitis patient who is agitated?
b. Assess the trachea and attempt to move Keep the light low
the trachea
c. Clamp the 2 chest tubes 94. Diet for Pregnancy Induced
d. I forgot Hypertension? Maintain well balanced diet
84. Why is there thirst on DM patients: 95. To prevent post-op Thrombophlebitis. The
A. Increase glucose triggers thirst, water is needed to patient is instructed to do which? Elevate with 2
dilute the sugar pillows
B. Increase glucose, increase sodium, excreted in
the urine, triggers thirst
96. enema adm. Left side knee flex
C. Forgot
D. Forgot 97. Purpose of emptying bladder before
paracentesis? to prevent puncture of the bladder
85. What is true on all DM cases?
a. Increase sugar is due to too much or too little 98. Ventricular Septal Defect sound heard? harsh
insulin produced heart murmur
b. In DM there is too little ketones that's why there is
ketoacidosis 99. which patient will alert for care ? patient with 25
c. Forgot ml/hr urine output
d. Forgot
100. Patient complains of abdominal cramping during
86. Pt being treated for ALL. Prone to infection this is colostomy irrigation, what to do? Slow the infusion
due to:
a. WBC cell is not responsible for fighting 101.Parameters of CGS, EXCEPT
infection it's the RBC instead a. pressure response
b. The pt has inc number of WBC that is not enough b. verbal “
to fight infection c. motor "
c. WBC is increased but they are not mature d. eye "
enough to fight infection

87.Sweat Chloride Test — Cystic Fibrosis

88. A nurse is ACCOUNTABLE for?


a.planning treatment and judging the effectiveness of
the interventions
b. planning tx and designing effective NCP
c. planning tx and judging the effectiveness of the
NCP

89. In Pancreatitis who is the responsible of the


multi organ damage and responsible for
autodigestion?
A. Amylase
B. Lipase
C. Trypsin
D. Cholecystokinin
REVIEWER – 2
Outlines-HAAD exam-March 2021

1. After tracheostomy, watch for patient keep on swallowing--sign of bleeding


2. Instilling medication on OS --left eye
3. Medication given prior thyroidectomy (not sure with my answer, please search the
right answer)-- choices i remember: Iodine, Propylthiouracil (PTU)
4. Signs of symptoms of a child with hydrocephalus, except--cannot remember the choices.
5. What is a dry chemical fire extinguisher suitable for, except--not sure with my answer,
please search for the right answer. (in google search, it is for wood, paper and electrical,
etc.)
6. Differentiate Spina Bifida Cystica from Spina Bifida Occulta
7. Definition of positve Coomb's test
8. Familiarize medications for Schizophrenia, Parkinson's and Alzheimer
9. Know the normal Potassium level.
10. Definition of pain--cannot remember the choices.
11. To familiarize the NPH and regular insulin administration.
12. To familiarize the rule of nines for Burn
13. Tetralogy of Fallot, all types of defects are correct, except--Transposition of great vessels
14. Lab values elevated in Pancreatitis--Amylase and Lipase
15. How you give ear drops to an infant--my answer but not sure if correct, pull pinna down
and back and direct solution onto the canal wall.
16. Normal range of central venous pressure (CVP)--cannot remember the choices (2-6 mmhg
per google search)
17. Important lab values to look at in a Sickle Cell Crisis disease--choices i remember,
hematocrit and hemoglobin.
18. When pain relieved in Duodenal ulcer --pain relieved by a meal-per google search)
19. One of the proper methods in tracheostomy suctioning--apply suction max of 10-15 seconds-
per google search
20. Is the HIV and AIDS the same? --cannot remember the answer.
21. Review basic information related to APGAR scoring.
22. What to monitor in congestive heart failure CHF-cannot remember the answer.
23. What to advise patient after hip surgery--cannot remember the answer.
24. Which pain assessment suits for children--cannot remember the answer.
25. Deep Vein Thrombosis-things to do--cannot remember the choices, i answered- do no
massage- but not sure with my answer.
REVIEWER – 3
3â2 .Patient with diabeti; peripfi eral ne uropa thy, what you will instruct? Check the temperature 0 f
the wann water with thermometer before bathing.
35 3. You are doing resea rch in your area, wha t you will con sider first? Pmvious results done by
somebody else.
354. Disease happened to glo mer\4ar membrane will re suit in wh at? PROIET NURTA
355. Patient received from PACK , immediately after receiving the patient
hecome agitated and restless? What 1s the nursing action? CHECK conscious level and check
the lair pain medication received
386. How wilt you diag lose type 2 DM? Fasting 6lood sugar >I20mg/dI
357.new wiT\ you maintain :nfecfiion control In post- partum pacierit? Wash hands before and efter

35a'. Which pulses are easily accesslbks7 RAOIAL and CAROTIO


59. In cfiiTarrrydiaT infection what you will suspect? CFRVICItIS
3d0. Patient came to hospiFal with inzectén, at night he became confused, agitated and disoriented,
What ¢ouTd be the reason? DELIRIUM
361. Colostomy patient encourage what iood? Crackers, toast, yogurt (low fiber)
36z. Pain assessment with a 3 year Old child who is grimacing use? WOflG aAKER PACE SCALE
^3S3.Orinase (totbt tamide) what is the contraindication? MAO INHIBITOR
36d. How to secure swab from fungal infection? Use a DACRON tipped swa6
365. Claucoma — LOSS OF PERTPHE RAL vlsloN
366. Post cataract surgery - PAtCH EYES DURING NIGHT
367 wa› »* » am» a rs, a ! acre press are afferfe‹i* A‹SESSMr N
368. A6G in asthma patient — RESPIRATORY ACIDOSIS
369. Actio n of SAL6UTAMOL in a$thrYja pstien r? RELAX in uscTes of bronchi
70. Dletary rnoditlcation of DM patients take into consideration? PATIENTS PREFERENCE OF FOOD
371. Where can you best hear the apiCa | pulse — 5" interco stal space, nn id-C1av'cuIa r a roa
372. Preserve ulcer — REPOST6ON PATIENT EVERY 2 HOURS
A73,BuIema — Controlling behavlor
374. What position indicated for patient aRer surgery for perforated appendix with localized peritonitis?
sEui-FowLEfi'› POsiTION
37S,Which of tha disease needs Airborne precaution? M£ML£S Auto V 8icEHA
376. Patient with fluid on the chest what sound? CRAC KLE5
373. Tn Breast cancer patient ERP (E5tpc+gen React?e Prote• I is posicive, what does it mean? DfifA
BONDING
3 7g, Drug that wlll Increase blood sugar level? 3tooT softener (nut sure utf er choices,
betabIocker,ACE INHIBITOR, diuretics)
37J. When a patient is w‹th urinary retention. how you will assess? PALf'ATtON
3&3. When the patient is unresponsive, in a standard cardio•puImonary arrest what i$ the immediate
intewention?CALL THE EMERGENCY RESPONSE TEAfvt
38a. To preserve communication ability fo a PaAinsonism patient, What t5 the nurse's ac\ion? Teach
to Do facial exercises suC{\ a$ 5mIIing and frowning382. Purpose of fiiGT wiFh patient e‹itI\ intestinal
b tructiori* To decomeress air and fluid from the stomach.
384. A senior nune who i5 mg nto ring a )un for nurse, ask the |unior nurse to insert NGO whic li she will
be doing the first time Who will be responsible? SENIOR NURSF
No r patie nt with te in p9 rary g acem aker who is going for surgery, which eq u ipme nt to ac¢0 m pany in
OR? ECG MONiToR
3ss. patient tells she Ie el s dizzy while the nurse Is passing in the hallway, what will be your nursing
B¢tlO D? MAkE THE PATIENT SST, do not leave patient
87. Which food 1s rich in I ron? LFTTUCE

Patient with depression, what is the assessment for him? HOPELESSNESS and HELPLE5StdŁSS
392. When you will consider patient with TB on treatment In\proved? SPUTUM AFB NE GATTVE
393. lnsufln fMixtąrd/NPH) taLen frorn the fridge, what y0u will do befure preparing? RDTATf BET.

3 94. Streptokinsa contrąindication? HYPE RTE NAION


3 9S. flidney b‹opsy position? PRONE POST - SUPINE
9s. eatient re.ceiving warfarin and KNR IS 3 what to do? GIVE THE WARFAPIN
397. Patient on warfarin gJth PT of 35, nursing asion?
398. For vi\ertt patients — ASSIGN ROOM HEAR NURSES' STATION
is given 0ESMOPREKIN/VA$OPRESSTH WHY?
4d0. For h*p repTacemeot patlen r w hat to provide* HIG H TOILET SEAT
4Li1. Before pre-medication the patient claimed the pm.dare not clear to fi'er, what \o do?
And I nfc rm
Raise side rails up

407. After head trauma pat'ent spent long time ir hospital, tf+a nurse should teach moral suppor to tne
family because — the family becomes dysfun¢t?onaT and needs suppor they have role in the tx cyCfe of

Wñm exeubat g /W parénr W ' I'M \mw air she > not lit for e•tubatjon? ISO
gA£ATH SOLINOS, UIF FULTY BREATHING, SECRETIONS
When giving medicaFion — CHECK NAME eaN0 if photo ID in the file is not present
410. in whiCg rate you see lncreasefl sodium?

PSyEho-behavi0raJ therapy for pain-advantaga? Pteąse stuóy


411 jałiem is ro take 2mg/ hr rnędicine, his weight is 70 kgs how ma ny mgs she will
take7 4t4. When you have high preCaution — use gIov+s when tcuching body I1uirIs
415. There is a new article about dressing.Nsg. Intervention — Inform the cha+ge nurse and discuss in the
ng wit
41a. Narcotics are locked - to avoid Mf5U5E
- the nurse applies the pads in — right he low clavicTe; Jeff in precordium
Premature I›abies appearance -THIS WASTED APPEARANCE

420. Time Management means — TI is a techniq a designed to assist in c0 mpleting task in a definitive

with Ifeosto
T28. Doctor ordered 5€iomg dopamine in s00 ml —available stock 2O0mg/5mI, how mpiy m1/hr to give?

Di oxi ^c•ease myoca


42B. Action of beta-blocker - Va sgdiłafion/ decreese BP
Dittiazem — CA CHANNEL BLOCKER
4 3tI. 80 yeah of0 a dmitte d to wa rd wh at jS the prio rity care tor satety? Teacł› how to u se the CAŁL gE LL
ADAĄWT PAlN ’ . .. ’• ’
432. Patient on Ałdactorte prone to - hy r\.,. .+’.• .:. ’
.
43*. Color”and odoréf.wounds indicates — PHASES Or wOunO HEALING ’
454. PPD TEST — FOR TB
455. Insulin given by tubercuTrne syringe 20 units give? 0.zmI
36. What is 1HD (\SCHEM IC HEARI DISEASE)describes a s — deposits of L1PID CONTACT ING PIQUE
I is tLe use of draw sheet Wtth patient on skeletal traction? Prevent shearing of the

138. After laminectomy — check lower extremities for pulse


439 CelTulitis/edema on deg - PRIORIW - £ levate the legs
Oigoxin was ordered 12Sm@ god Stock is .zs0mcg/tab. How mu¢h you will give? y tab

NPH give« at 8am what time is the peak of action?

Hyperemesis gravidarum - Mecabolic alkaTosis


445. Fra eta re of acetabulum which part of the tern nr is affected — HEAD
446, Pa tient for CT SCAl\j - ask f0r a1Iergy to seafoods
447. Ham Bn's sign — DVI {calf pain en dors‹flexion of rhe foot)
0s 2 affectr - uii0Dt-E AGED PEOPLE
449, 8efore prep9rlng tr administer drug — CHECK PHYSICIAN S ORDER
Oxygan fi considered dangerous cause — EXPLOSIVE
After enal biopsy rvhap should not be done? AVOt0 STItENOUS ACTIVJTY
Patient with Canr.er tetminal staga, I+a says that "I want to die today, I don't want to live anymore,
that is tfie coping mechanizm?
ng ge
454. After givirig IM Injection - if bleeding occurs, APPLY GŁNTŁE PRESSURE FOR RMETIME
455. A nurse was seen taking medicine from the cabinet what is your nurs‹ng action? Talk to the nors
te II her to return medicine or you will irform the supervisor
56. Emphysema - Destruction of ĄLVEOLAR WALLS
REVIEWER – 4
15. > SATA: an infant of a diabetic
1. > Q:What is the stimuli for respiration ?
A: Carbon Dioxide mother a.full face
b.fat
c. hair in the outr ear
4. > Q: Sequence of abdominal assessment
A: inspectiom, auscultation, percussion, papitation 16. > signs and symptoms of a
child with hydrocephalus
5. > Q:After tracheostomy,
what to assess?hoarseness

6. > Q:What is the best position of the client


after liver biopsy.
A: Answer niya right side with pillow under site 17. > tetralogy of fallot, all is included
except? Transposition of great vessels
7. >What is the temp of the sol of enema in
children? 18. > A 3oé solute, 60mg solution
A. 90 o F
B. 100 o F a. 2mI
C. 105 o F ? b. 2.5
D. 110 o F c. 5mI
d.20
8. > Q:In instilling medication “OS” what
does “OS” mean? 21. what meds are taken sublingually?
A: Left eyes
a. lipitor
9. >What do you expect to be ordered for b. Nitroglycerine
depressed patients? c.HaIdoI
A. SSRI d.
B. ECT
C. St. John's wort 22. > involves absorption, distribution, metabolism,
excretion of drugs
10.>What is akathisia?
a. Pharmacodynamics —biochemical and physical
11. > Patient tell the nurse that CIA is effects of drugs and the mechanism of actions
looking for him and wants to h. Pharmakokinetics
Kill him. What would be the best nurse response? c. Pharmacology scientific study of origin, nature,
chemistry effects, and uses of drugs
d.Pharmacotheerapeautics- use of the drugs to
12. > Patient was rushed to the emergeny room prevent and to treat
due to car accident.
What would be the best to assess 23. What is the purpose of nursing licensure?

a. When was the last intake of alcohol?


b. What kind of alcohol was ingested? Dry chemical Fire estinguisher for what type of fire
c. whats the alcohol blood concentration?

Spina Bifida Occulta


14. > A child swallowed marble. What would
indicate that the child is
SATA .select answers that have “research”
having total obstruction?
Cerebellar injury, what will be affected
a. coughing
b. gagging
chest pain is increased/ triggered by co/c/ wafer
c.tachypnea
>reIaxation therapy effective for emotionally unstable
client except
>exercise
>guided imagery
>progressive muscle relaxation

>1 question in rule of 9 ( right arm and right leg)


>7 months pregnant complains of bleeding gums, this
may be related to
increase in progesterone

evidence based practicenursing research


quali and quanti research
diabetes type 1

presbycussis
below knee amputation
calcium daily mg intake requirement
REVIEWER – 5
• Positive coomb's test
• Peritoneal dialysis — when to call the • 88 yrpt with poor nutrition has
dr. if the drain is cloudy- colorless- ulcer what is the cause beside his
pink- strew poor nutrition
• Obstructed pile: dark urine and • Cognitive thoughts! How it help to
pale stool reduce pain?
• Tracheostomy pulsation and bleeding • NPH and regular insulin administration
what to do? • BSA + child dose
• Nasogastrolarengeal suction and • Rule of nine
oropharyngeal suction comperation • 8 month baby fall down of his bed
• Meds for schizophrenia Parkinson's and came with vomiting what other signs
Alzheimer to call the dr.?
• When the nurse cant hear the heart • Pt hadvirtectomy with drain what is
sound what to do: position the the risk factors?
patient on his left side, on the right • foramen ovale
or let him left his hand up? • femur + buck traction
• S1 not heard clearly in which case? • nurse interviewing pts to complete
• Dry mouth, dehydration with high her research what is the type of
potassium level what to give? N/S research she is doing?
• Potassium level: 3.5-5.3 or 3.4-4.7 • Important competency for clinic
• What precautions to keep for pt nurse to do?
with meningitis? Enteric- • When to use warm technique to reduce
preventive- respiratory? pain
• You see in pernicious anemia pt • VT treatment
except: glositis- jaundice- parestesia? • Pt with pancreatitis what to expect?
• Types of headache Bright color line? Discoloration
• How many kgs a pregnant woman around abdomen? Edema
d d
gain 2’ and 3’ trimester? periumbilical? Pain lower left
quadrant?
• What restrains used to 4-6 month
infant? • Pancreatitis labs results
• ABCD of skin cancer • Drop factor calculation
• Facial nerves • Pt dehydrated how to assess during
iv fluid is administering
• Pt with glaucoma take clozapine?
What are the side effects? • 2 eye drops ( antibiotic and
prednisone) all right except:
• What is cataract
refrigerate the drops- shake before-
• What conditions reduce the neutrophils
clean the eye — give them separated 5
count
min apart
• Why WBC increase in ALL pts
• How u give ear drops for 6 yrs old
• Priorities in ER assessment
boy: give be4 swimming- warm the
• Types of spina bifida
solution
• What instructions you give to
an asthmatic child's parent
before discharge?
• We do dressing why? To keep warm- • Increased ICP signs
to keep moist- to keep dry- to
prevent necrosis?
n
• 2 d prevention

st
1 prevention
• How to do dressing for injured
hand? From dorsal to proximal-
with 25 degree fingers flexed?
• Types of dressing for ulcer
• What antibiotic not to give for pt
with allergy to penicillin
• Fefol complications
• High glucose level with ketones
urine what is the diagnosis?
• Emphysema signs that the nurse
can assess?
• Esophageal varices which condition
cause it? Cirrhosis- CHD
• Asthmatic child came to er with SOB
and nasal flaring what is the
priority? administer 2 puffs ventolin-
assess o2
• Pt with previous COPD came to er
with chest pain and sweating what
is the priority? Give nitroglycerin
immediately — check RR and 02
• Pregnant women came with gum
bleeding, the nurse know that this
is due to?
• What is the most common
oncologic disorder and the most
curable one?
• While administering enema for a pt he
complains of abdominal cramp, this is
becoz? The solution is cold- hot- its
too fast
• Most common pt to have fluid
deficit: post op pt 3 days ago with
ileostomy
• Cranial nerves
• Patient with head injury what to
expect
• Early signs of pregnancy
• Pain?
• Progress notes?
REVIEWER – 6
1. Manifestation of HYPOXIA?

2. Manifestation ng HYPETHYRODISM? Hypersecretion of T3 and T4, S& Sx enlarged


thyroid gland, palpitations, cardiac dysrhythmias such as tachycardia or A. Fib., protruding
eyeballs exopth, hypertension, heat intolerance, diaphoresis, wt loss, diarrhea, smooth soft skin
and hair

3. ABRUPTION PLAcenta?SATA

4.WHO?

5.PRIVACY?

6.Cont.Education y kailangan ng NURSES?

7.Evidence base practice?

8.Dossage and solution computation (BASIC)

9.Management in depresse pt?

10. needs airborn precaution?


SATA a.TB
b.VAriceIIa
c.rubella

11. Precaution in meningitis?


A,enteric
b.respiratory
c.isolation

12. Lack of intrinsic factor? PERNICIOUS


ANEMIA a.sickle
b.macro?
c.aplastic

13. OK-OK
Managament? 14.End
stage renal failure
a.inc.ph
b.hypercal
c.inc.BUN
d.inc Crea

15. Im dying ,pt tell to d


nurse? a.ignore
b.sit wit pt,ds must b difficultym 4 u
c.teeI sa family

16. Rich in Tyramine


a.soy sauce
d.cheese
c.fruits
d.
17. Meniere
disease a.safety

18.CPR
A,15:1
b.30:02:00

19. MS fP screening ba un?


a. inc and dec result in abnormalities
b. AFP secrete
by liver c?d?

20. during seizure wat position para daw ma ensure na ang airway
ay oky? a.lateral
b.prone
csupine
d.hyperfIex d neck

21. NGT PURpose?


22. HEAD lice meds?
23. error in doc wat
to do?
24. purpose of progness
note?
25.BT?SATA
a.get sa blood bank 30mins bago I
hook b.tell to pt to void
c.hook 5% water

26.beta blocker to tx high blood pressure?


A,tenormin

27.cataract?
28,ECT para san?
29.liver biopsy position
30.ear drop instruction?
31.Dx test for
Diverticulitis? a.barrium
enema b.colonoscopy
c.CT
d?

32. niIagyn ng drain ang skull after surgery high risk daw
ang pt sa? a.infection
b,ICP
33. PeritoniaI dialysis
repor?
A,cloudy

34. lab result ng


pancreatitis? 35.cluster
headache 36.aminophyIIe
SE?SATA i.consti
iidrymouth
iii.tachy

POST KO NALNG PAG NA ALALA AKO HA!!sensya na „d pa gumana utak sobra depress!!!!
REVIEWER – 7
1. An unconscious patient is admitted because of
head injury secondary from vehicular accident, he is known as alcoholic, what
is the nursing priority during emergent phase of admission?

a. perform jaw thrust maneuver

b. assess location

c. check alcohol level

d. determine the type of alcohol ingested


2. The first priory after insertion of tracheostomy tube is?
a. maintaining strict asepsis

b. prevention of bleeding

c. preventing any obstruction to tracheostomy tube


3. Lithium toxicity value

A. 0.5-1.0,
B. 1.5-2.0 (2013 hogan)
C. 2.0-3.0,
D. 3.0-50
4. Lead toxicity value in child?
A.2-5
B.5-10 (2012 guidelines}
C>10
5. Dry mouth, dehydration, with high potassium level.what should administer?
A)lactated ringer som
B)NSS
6. Sign of emphysema:???
A) cough
B) tachypnea
7. What solution makes fluid to go into the cell?

a. Isotonic

b. Isometric

c. Hypertonic

d.hypotonic
8. A nurse reads in progress notes for a client with pneumonia that areas of the clients lungs being
perfused but are not being ventilated . the nurse interprets as this concurrence as the presence

A. Anatomical dead space


B. Physiologic dead space
C. shunt unit
D. Ventilation-perfussion matching
11. Tan skin in patient with Addison is caused
by

a. overproduction of corticosteroids
b. underproduction of corticosteroids

c. overproduction of ACTH (The pituitary sees that drop in cortisol levels and responds by making
more ACTH. ACTH is derived from a bigger precursor molecule called pro-opiomelanocortin (POMC). POMC is
also a precursor for beta endorphin and melanocyte stimulating hormone (MSH))

d. underproduction of ACTH
12. Why is there a need for
licensure exam?

A. To ensure minimum knowledge

B. To ensure excellent knowledge in


specialized area

C. To assess if
nurse is fit to practice

D. To assess continuing practice


13. Patient female has undergone induction therapy for leukemia. The ff are precautions intructed
to the pt:

I. Use contraceptives to produce amenorrhea


II. Avoid use of dental floss (The bone marrow depressant effects of doxorubicin may cause an
increased incidence of microbial infection, delayed healing and gingival bleeding. Patients should be instructed in
proper oral hygiene during treatment, including caution in the use of toothbrushes, dental floss and toothpicks.
Dental work, whenever possible, should be completed prior to initiation of therapy, or deferred until
blood counts have returned to normal.)
14. For FBS test, advise given to patient is:
A) take lot of fluid before day of test
B) take small meal at 6am and do test at 10am
C) avoid smoking for 1week
15. In which of the ff condition/s S3 is heard softer.
SATA I.Hyperthyroidism,
II. heart failure,
III. mitral regurgitation,
IV. mitraI stenosis

16. In aging clients, which anatomic changes requires additional measures:


A. Difficulty in swallowing
B. Decreased sensation of pain
17. Assessment after tracheostomy —
a. Drop in BP ( Tension Pneumothorax/ bleeding )
b. Hoarseness of the voice ( myt suggest laryngeal damage or bleeding too)
18. Which of the ff is true with transfusion of FFP and Platelets?
A. FFP needs ABO compatible and Rh compatible
B. Platelets needs ABO and Rh compatibility
C. FFP needs ABO but not Rh
D. Platelets needs ABO but not Rh
19. What hormone stimulates the release of bile to the mesentery?
0 cholecystokinin

20. Urine collection, what should the nurse consider?


A. Sterile container
B. Nenstruating women may not access accurate result
C. Strict aseptic technique
D. Time of collection

21. The patient is being treated for amphetamine abuse. His friend visited him and said he changed
his behavior and becomes euphoric. Which is an appropriate nursing action:
A. Search room for drugs
B. Ask him if he has taken drugs
C. Check for urine drug test
22. Pt with leukemia, upon his 1st chemotherapy he had sign of nausea vomiting constipation and
dyspnea. nurse knows next scheduled chemo regimen,pt will be:
A)eat light meal before therapy
B)will receive antiemetics 1hr before tx
23. A nurse is teaching a young adult to be aware of common cause of morbidity:
A. Accidents
B. Respiratory problem
C. Cardio problem
D. Gastrointestinal problem
24.Situation: Matthew has a compound fracture of the temporal bone.

1. The nurse notices bleeding from the orifice of the ear. Which of the following
actions by the nurse can be safely used to determine if the drainage contains cerebrospinal
fluid (CSF)? The nurse should:

A. swab the orifice of the ear with sterile applicator and send the specimen to the laboratory
B. blot the drainage with a sterile gauze pad and look for a clear halo or ring around
the spot of blood
C. gently suction the ear an send the specimen to the laboratory
D. test the CSF with a Test-Tape and get a negative reading for sugar

2. The nursing care plans states —Observe for early signs of increased intracranial
pressure (ICP). Early symptoms of ICP include:

A. widening pulse pressure and dilated pupils

B. rising blood pressure and bradycardia


C. elevated temperature and decerebrate posturing

D.nausea, vomiting, and restlessness

3. During the initial period after a head injury, nursing intervention for Mr. Matthew
should include :
A. packing the ear with cotton balls to stop bleeding
B. awakening the patient every 2 hours to determine his level of consciousness
C. placing the patient in Trendelenburg's position
D. forcing fluids to restore hydration
25. Pt is having dyspnea, diaphoresis, cold clammy skin and with rapid pulse and irregular who had
previous CHF and not ever had MI:

1. Which of the following is a key assessment finding for the pt diagnosis:


A. Neck vein distention
B. lower extremities edema
C. Hepatomegaly
D.Crackles
2. Which of the ff would the nurse to anticipate to be the physicians priority:
A. Hooking pt to cardiac monitor
B. Order ECG
C. Maintain IV line
D. Taking pt's history
3. The patient is given diuretics, which of the ff would be a priority assessment when the
drug is taking effect:
A. Heart rate
B. Respiratory rate
C. Urine output
D. Electrolyte levels
26. Situation: Henry is a 43-year old man who has been rushed to the emergency room
with an acute gouty arthritis.

1. While admitting I‘m. H. to the hospital, the nurse should recognize those factors that can
precipitate an acute attack. They include :

A. excessive smoking
B.large alcohol intake
C.emotional stress
D. improper rest
2. A serum uric acid level is performed by the hospital laboratory. In acute gout, the uric acid
level is approximately
A. 1.0 mg/100 ml
B. 2.1 mg/100 ml
C. 6.5 mg/100 ml
D. 10 mg/100 ml
3. The expected outcome for colchicine drug used to treat acute gout is to :

A.reduce uric acid levels


B. relieve joint pain and inflammation
C.increase blood flow to the kidney
D.detoxify purines in the liver

4. During the night, Mr. H. complains of severe pain in his toe and asks the nurse for 2
aspirin tablets. The nurse should:

A. give the patient the 2 aspirin tablets


B.elevate the foot on a pillow
C. notify the physician
D. offer the patient a cup of tea

5. Some physicians prescribe an alkali-ash diet to enhance the effect of the medications. Which
of the following foods are allowed?

A.liver, shellfish, and fats


B.cranberries, cheese, and whole grain cereals
C. milk, vegetables, and most fruits
D.eggs, milk, prunes, and plums

27. Mrs. Hashim is a 50-year old woman who has a spinal cord lesion at the fourth thoracic
(T4) vertebra.

1. When there are lesions above T4 and T6, the patient may experience autonomic
hyperreflexia. This condition can be prevented by :

A. avoiding bladder distention


B. changing the patient‘s position hourly
C. wearing supportive elastic hose
D. doing a neurologic check
2. Mrs. Hashim complains of severe headache and is extremely anxious. The nurse checks her
blood pressure and finds it is 210/110. The nurse should then :

A. check the patency of the urinary catheter


B. apply ice packs to her head
C. place the patient in a flat position
D. sit with the patient until the symptoms subside

28. Pt receivng antipsychotics develop head and neck stiffness wat actn shud take?
A)admnstr anticholinergics
B)administr cholinergics
29. Pt with vomiting and diarrhea cause:
A)hyponatremia
B)hypernatremia (N/V is signs and symptoms of increased sodium)
30. Which of the ff is true with transfusion of FFP and Platelets?
A. FFP needs ABO compatible and Rh compatible
B. Platelets needs ABO and Rh compatibility
C. FFP needs ABO but not Rh
D. Platelets needs ABO but not Rh

31. Which is more prone to nosocomial


infection? A)40yr pt with tracheostomy
B)2yr baby with 2nd degree burns 80a/a of body
32. A client admitted to the hospital with severe with loss after extreme dieting. the nurse plans care
knowing that which of the following physiologic processes occurs in a prolong absence of
adequate food intake?
A.Glucose metabolism
B.GIycogenoIysis
C.Gluconeogenesis
C.Lactic Acidosis
33. Obese client with bedsores, what is the priority nursing diagnosis:
a. ineffective health maintenance
b. imbalanced nutrition: more than body requirements
c. risk for infection
d. impaired skin integrity
34. Sickle cell anemia pts should avoid:
I. Extremes of temp (ALL can cause I o2)
II. High altitudes
III. Strenuous activities
35. Samantha is a 25-year old patient who has experienced increasing generalized stiffness,
especially in the morning, fatigue, general malaise, and swelling and pain in the finger joints. She
has a tentative diagnosis of rheumatoid arthritis.

1. Which of the following blood-analysis tests would be consistent with diagnosis of


rheumatoid arthritis?
A. an elevated erythrocyte sedimentation rate and negative C-reactive protein
B. an elevated erythrocyte sedimentation rate and positive C-reactive protein
C. a low erythrocyte sedimentation rate and negative C-reactive protein
D. a low erythrocyte sedimentation rate and positive C-reactive protein
2. The primary goal of nursing care for Mrs. S during this initial acute phase of
rheumatoid arthritis should be to :
A. prevent deformity and reduce inflammation
B. prevent the spread of the inflammation to other joints
C. provide for comfort and relief of pain
D. assist her to accept the fact that rheumatoid arthritis is a log-term illness
3. During hospitalization, the nurse should explain to Mrs. Samantha that analgesics of choice
would be :
A. codeine
B. acetylsalicylic acid (aspirin)
C. acetaminophen (Tylenol)
D. proppoxyphene hydrochloride (Darvon)

4. During the acute phase of Mrs. S‘s illness, which of the following measures would be
the most appropriate?
A. frequent periods of active exercises
B. frequent periods of bed rest
C. rest for he affected joints only
D. encouragement to perform activities of daily living independently
36. Commonest sign of HTN:
A) headache
B)epistasis
37. An old woman was hit by a motorcycle, The old woman told Kareem that she has osteoporosis;
Kareem knew that all of the following factors would contribute to osteoporosis except:

A. Hypothyroidism
B. End stage renal disease
C. Cushing‘s Disease
D. Taking Furosemide and Phenytoin.

The old woman was now Immobilized and brought to the emergency room. The X-ray shows
a fractured femur and pelvis. The ER Nurse would carefully monitor her for which of the
following sign and symptoms?

A. Tachycardia and Hypotension


B. Fever and Bradycardia
C. Bradycardia and Hypertension
D. Fever and Hypertension
38. SITUATION: Agnel, A male patient diagnosed with colon cancer was newly put in colostomy.

1. Agnel shows the BEST adaptation with the new colostomy if he shows which of the following?

A. Look at the ostomy site


B. Participate with the nurse in his daily ostomy care
C. Ask for leaflets and contact numbers of ostomy support groups
D. Talk about his ostomy openly to the nurse and friends

2. The nurse plans to teach Agnel about colostomy irrigation. As the nurse prepares the
materials needed, which of the following item indicates that the nurse needs further
instruction?

A. Plain NSS / Normal Saline


B. K-Y 3eIIy
C.Tap water
D. Irrigation sleeve
3. The nurse should insert the colostomy tube for irrigation at approximately

A. 1-2 inches
B. 3-4 inches
C. 6-8 inches
D. 12-18 inches

4. While irrigating the client's colostomy, Agnel suddenly complains of severe cramping.
Initially, the nurse would:

A. Stop the irrigation by clamping the tube


B. Slow down the irrigation
C. Tell the client that cramping will subside and is normal
D. Notify the physician
5. The next day, the nurse will assess Agnel's stoma. The nurse noticed that a prolapsed
stoma is evident if she sees which of the following?

A. A sunken and hidden stoma


B. A dusky and bluish stoma

C. A narrow and flattened stoma


D. Protruding stoma with swollen appearance
6.Agnel asked the nurse, what foods will help lessen the odor of his colostomy. The nurse
best response would be:

A. Eat eggs
B. Eat cucumbers
C. Eat beet greens and parsley
D. Eat broccoli and spinach
7. The nurse knew that the normal color of Agnel's stoma should be :

A. Brick Red
B. Gray
C. Blue
D. Pale Pink
39. After mastoidectomy, Nurse Kareem should be aware that the cranial nerve that is
usually damage after this procedure is

A. CN I
B. CN II
C. CN VII ( The facial nerve branches from the back of the ear and spread towards the mouth, cheeks,
eyelids and almost all over the face. In mastoidectomy, Incision is made at the back of the ears to clear the
mastoid air cells of the mastoid bone that is infected.)
D. CN VI

40. If Mr. Samuel is receiving pilocarpine, what drug should always be available in any case systemic
toxicity occurs?

A. Atropine Sulfate
B. Pindolol [Visken]
C. Naloxone Hydrochloride [Narcan]
D. Iflesoridazine Besylate [Serentil]
41. The doctor orders timolol [timoptic]. Nurse 3ames knows that the action of this drug is:

A. Reduce production of CSF


B. Reduce production of Aqueous Humor
C. Constrict the pupil
D. Relaxes the Ciliary muscle

2. Nurse James knows that the normal IOP is :

A. 8-21 mmHg
B.2-7 mmHg
c.3135mmHg
D. 15-30 mmHg

3.When caring for Mr. Andrew, James teaches the client to avoid :

A.Watching large screen TVs

B. Bending at the waist

C.Reading books

D. Going out in the sun


42. Ear drops are prescribed to an infant, The most appropriate method to administer the ear
drops is :
A. Pull the pinna up and back and direct the solution towards the eardrum
B. Pull the pinna down and back and direct the solution onto the wall of the canal
C. Pull the pinna down and back and direct the solution towards the eardrum
D. Pull the pinna up and back and direct the solution onto the wall of the canal
43. The patient was prescribed with levodopa. What is the action of this drug?

A. Increase dopamine availability


B. Activates dopaminergic receptors in the basal ganglia
C. Decrease acetylcholine availability
D. Release dopamine and other catecholamine from neurological storage sites
44. You are discussing with the dietician what food to avoid with patients taking levodopa?

A. Vitamin C rich food


B. Vitamin E rich food
C. Thiamine rich food
D. Vitamin B6 rich food
45. Nurse teaching parents of a child with heart failure. The family is correct when they state to:
A. Administer diuretics, monitor weight and I and 0
B. Admin diuretics, admin potassium, sodium restrict and monitor land 0
C. Sodium restriction, restrict fluid to 100cc, monitor I and 0

46. Alzheimer patient unable to sleeps at night what is the management?


A)physical exercise at day time
B)sedat
e
C)ignore
47. When feeding a patient with gastrostomy tube, which of the following intervention should be
the nurse's priority?
A.Elevate HOB at least 30•
B. Initially flush the ostomy with
30mI water C.Check patency by
auscultation
D.heat formula in room temperature
48. Which situation should have standard precaution other than using gloves?
a. barium enema administration
b. interstitial radiation on oral cancer
c. renal arteriogram
d. vomiting patient on brachytherapy
49. Nursing responsibility in the 3rd stage of delivery, especially in the expulsion of placenta
A. Mild fundal push
B. May occur up to 3 hours
after infant delivery, wait
patiently
C. Administer oxytocin, IV or IM as
bolus
D. Ask pt to bear down slightly
50. Icp normal value?
A. a-7-12 mmhg
B. b-5-12mmhg
C. 12-15mmhg
D. d-12-20mmhg
51. Which piece of information from the client's medical history does the nurse determine as a risk
factor for developing osteoarthritis?
A. viral conditions
B. occupation
C. genetic predisposition
D. neuromuscular dysfunction

52. What causes irritability in a patient with cast?


A. Pain, immobility, dietary restriction
B. Immobility, edema, itchiness
C.Pain, edema, inability to do adls
D. Fluid restriction, pain, immobility

53. Normal respiratory stimulant in COPD:


A. CO2
B. hypoxia
54. Which help nurses in prioritizing plan of care?

A. Nursing care plan


B. doctor's orders
C. Nurse's progress notes from previous shift
D. kardex
55. Eschar that is dry and hard covers a foot ulcer should be:
A. Debrided to promote granulation tissue growth
B. Debrided to prevent secondary infection
C. Covered by soft dressings
D.Treated with hydrocolloid

56. Patient diagnosed with osteoporosis, what should be recommended?


a. bed rest with bathroom privileges
b. keep affected part immobile
c. weight bearing exercises
57. The ff should be done to confirm ngt placement except:
A. Xray of abdomen
B. Aspiration of gastric contents
C. Immersing ngt in the basin with water
D. Auscultating the abdomen while injecting air with the syringe
58. child with ADHD, showed inappropriate behavior on socializing with other kids
a. isolate patient for inappropriate behavior
b. let him explore more so that he'll realize his fault
59. Post cholecystectomy the nurse saw an oozing of blood in the dressing. The nurse should
immediately:
A. Change dressing
B. Mark with pen and check for change in size
60. Strict vegetarians are at risk to what type of anemia:
A. Aplastic
B. Plegaloblastic
C. Sickle cell
61. Patient post cholecystectomy is being discharged with t-tube in situ. The nurse should instruct:

A.Should have t-tube at level or above the site


B.Wear tight clothing to maintain t tube in place
C.Drain at the same time everyday

62. CEA is positive with?


I. Colon ca
II. Pancreatitis
III. Inflamatory bowel ds.
A. 1&2
B. 1&3
C.1,2,3
D.1 only
63. A dry powder extinguisher can be used for the
following: I-wood, paper, cloth
II- flammable liquids
III- Cooking oil
IV- Electrical Equipment
I only
I, II, III
III,IV
I, II, III, IV
64. High in saturated fats
I-beef
II-nuts
III-olive oil
a. I only
b. I, II
c. I, III
d. I, II, III
65. The patient run from a marathon has collapsed and brought to ER. Assessment of hypoglycemia
and ketones on urine is assessed, which of the ff is the cause.

A. DM
B. DI
C. Dehydration
D. Excess strenuous activity
66. A patient with abg result of7.45 ph, 30 co2, 25 Hco3 would suggest?
A. Respiratory Acidosis
B. Respiratory Alkalosis
C. metabolic Alkalosis
D. metabolic Acidosis
67. Patient with cholecytectomy who is obese is expected to be advised with what diet:

A. High fat diet


B. High protein to stimulate wound healing
C.Low carbohydrate
D. Low calorie to facilitate weight loss
68. Meds that can be used as rescue during bronchial asthma:
I.AIbuteroI
II. Ipratropium
III. Salmeterol
A. 1 and 2
B. 1 and 3
C. 2 and 3
D. 1, 2, and 3
69. Which of the ff patient needs intermittent urinary catheterizations
A. Female elderly with stress incontinence
B. Bladder training of patient with CVA (q 4-6hours)
C. TURP d. Patient with bladder spasm
70. A patient is having blood in the stool. Which diagnostic procedure is expected to be ordered:

A.FOBT
B.Colonoscopy
C.Sigmoidoscopy
71. In patient with Hepa A, least priority of the nurse would be:

A. Private room
B. Handwashing before and after
C.Using gloves when providing nursing care

72. A mother of a 13 year old female child expresses concerns about her child being chubby
and having fat deposits, the nurse should respond:

A. Normal for her


B. Increase exercise
C. Low fat diet to avoid fat deposits
D. Dietary restrictions

73. The nurse instructs the patient to prevent infection with which of the ff statements:

A. Eat foods from the 4 groups of foods


B. Take multivitamins when infection is present
C.Have immunizations after having an immunization

74. Patient with lupus should be instructed to do which of the ff.


I. Dry the body thoroughly by pat drying
II. Keep the skin dry by using powders
III. Use sunscreen protector with SPF of at least 30
A. I only

B. I and II

C. I and III
D. All of the above

75. A patient post bronchoscopy develops sudden dyspnea and shortness of breath. The
nurse suspects the pt developed pneumothorax, best test to confirm is:

A.X-ray
B.UTZ
C.CTscan
D.Bronchoscopy
76. A nurse has newly been assigned on a 12hr shift night duty which describes the needs for
rest and sleep

A. Attend exercise class on the way home


B. wear sunglasses when driving home and sleeping in a darklit room
C. Caffeine before the start of 12hr duty

77. Probe of the pulse oximeter can be placed in which of the ff if the fingers are not available

A. Earlobe
B. Nose
C. Eyebrow
D. Genetalia

78. Schizophrenia has becoming forgetful, I can't follow planned activities is related to:
A. Low IQ
B. inability to make simple decisions
C. inability to stock knowledge

79. During applying dressing in the hands with wound, the nurse should make sure the
following technique is to be done? SATA
I. Apply dressing from distal to proximal
II. Apply dressing with the fingers separated from each other
III. Maintain flexion of finger 25 degrees.
A. I only
B. I and II
C.IIandIII
D. All
80. A PICU nurse encounters an infant who just had tonic clonic seizure and the doctor tells
the nurse to give diazepam now. The nurse is aware that the exact dosage he should
administer if the infant's wt is 3.7kgs is:
Formula: Weight in Kg x 0.2 = 0.74 mg
A. 0.75mg
B. 1mg
C. 1.5mg
D. 0.5mg
81. Codeine to relieve pain of patient having severe headache would benefit if drug is given in what
route:
A. Oral
B. IV
C. IM
82. Jaundice during the first 24 hours of the infant's life?
a. physiologic jaundice
b. ABO incompatibility
83. Insertion of rectal suppository:
A. 1/2inch to the rectum
B. Let the suppository pass the sphincter of rectum

84. The ff are given to JRA except:


A. Naproxen
B. Aspirin
C. Ibuprofen
D. Tolmetin
85. All of the fT are contraindicated for forcep delivery except.
A. Face presentation
B. Incomplete dilatation of cervix
C. Cord prolapse
D. Breech
86. The nurse follows standard nursing documentation with which of the ff:
A. Using corrective tape
B. Recording patient activity even if insignificant
C. Using non erasable ink in documentation
87. Administered to prevent post partum hemorrhage:
I. Prostaglandin
II. oxytocin
III. Methergine
A. I and II
B. I and III
C. II and III
D. I,II, and III
88. "off the linen", what device is used:
A. Footcradle,
B. Air mattress
89. Which of the ff tech wud be beneficial for a pt with insomnia?
A. Guided imagery
B. Yoga
C. Muscle relaxation
90. A patient with DM 2 is treated with Metformin is to undergo CT- angiogram with dye. The
nurse should instruct the patient to:
A. Bring snacks to eat after the procedure because the pt fasts post-midnight
B. Adjust the Metformin before or after the procedure
C. Advise that he will experience blushing sensation as the dye is being injected
91. Patient came in the ER with penetrating blunt chest trauma. Priority nursing assessment would
be baseline hemodynamics?
A. Heart rate and respiratory rate
B. Standing and sitting BP
C. BP on both extremities
D. BP during inspiration and expiration
92. Mr Shaheed is recovering from myocardial infarction attends stress level management and
exercises what type of prevention
A. Primary
B. Secondary
C. Tertiary
D. Illness prevention
93. A patient with bacterial pneumonia receives chloramphenicol, which exam is priority assessment:
A.CBC (Serious and fatal blood dyscrasias (aplastic anemia, hypoplastic anemia, thrombocytopenia, and
granulocytopenia) are known to occur after the administration of chloramphenicol. In addition, there have
been reports of aplastic anemia attributed to chloramphenicol which later terminated in leukemia. )
B.PT
C.Liver enzymes
94. Which of the following would benefit from heat therapy:
A.Mastitis
B. Tooth abscess
C. Swelling knees with abrasion
D. Leg pain from strenuous exercise
95. You are caring for a client with end stage renal disease. Which of the following would you
anticipate to be seen in a patient that led to differentiate other renal disease manifestations with
end stage renal disease?
a. Anemia
b.Increase serum creatinine
c.Elevated blood urea nitrogen
d. Hyperkalemia

96. After change of shift, you are assigned to care for the following clients. Which client should you
assess first?

a. A 68-year-old client on ventilator who needs a sterile sputum specimen sent to the laboratory.
b. A 57-year-old client with COPD and pulse oximetry reading from previous shift of 90%
saturation.
c. A 25 year old patient diagnosed with asthma receiving zafirIukast(AccoIate).
d. A 21 year old patient receiving neostogmine(Prostigmine) due meds in 10 mins.

97. Immediate intervention for unconscious patient after vehicular accident.


a. 3aw thrust
b. Oxygen
c. Assess LOC

98. A client is admitted to the emergency department with an acute asthma attack. The physician
prescribes ephedrine sulfate, 25 mg subcutaneously (S.C.). How soon should the ephedrine take
effect?

A.Rapidly.
B.In 3 minutes.
C. In 1 hour.
D.In 2 hours.

99. A mother is 28 weeks gestation is for indirect combs test, what consider to be a positive?
A.Maternal serum of (+) antibody
B.Maternal serum of (-) antibody
C. Fetal serum of (+) antibody
D.Fetal serum of (-) antibody
100. Recommended daily allowance of calcium?
A.300
B.500
C.1500
D.3000
REVIEWER – 8
1. In DM client which drug would increase blood glucose?
a.Diuretic
b.ACE
c. Analgesic
d. Stool softener
2. What would be the nursing action for a patient on skin traction?

a.immobilization
b. range of motion exercises
c. make the patient to sit on a chair
d. ask the patient to dorsiflex the ankle
3. What is the common type of CA among children that has good prognosis?
A. Neuroblastoma
B. osteosarcoma
C. Leukemia
4. Which of the following factors should be the primary focus of nursing management in a patient
with acute pancreatitis?

a. Nutrition management.
b.Fluid and electrolyte balance.
c. Management of hypoglycemia.
d. Pain control.
5. with duretics administration, the nurse must be aware of:
A. high BP
B. weak pulse
C. muscle twitching
6. the first priority regarding medication administration ?
A. chceck pts name
B. check the expiry date
C. check physician order
D. check medication name
7. Before giving Digoxin, what Must the nurse do?
A. Assess the BP
B. Assess the RR
C. Assess the HR
D. assess the 02 saturation
8. MMR contraindication SATA
I.mild colds with no signs of
fever
II. allergy to
gelatin III.allergy to
neomycin
a. I and II
b. I and III
c. II and III
d. I, II and III
9. 12 wks pregnant rush to emergency hospital with vaginal bleeding. The nurse should know?
a. D and C
b. Use gram HCG to monitor er.
c. Spontanous abortion
d. instruct bedrest to reduce further vag. Bleeding

10. Room should be closed at all times. SATA


I. airborne
II. droplet
III. Contact

A.I only
B. IIonly
C.IandII
D.I,II, III
11. Which would decrease lymphocytes? (select all that apply)
I. Crohns Dse
II. Lymphoma
III. Hodgkins Dse
IV. Lymphotic Luekemia

12. A patient is asked to attend the medical research study, what protect the patient.

a.informed consent
b. nurses act
c. hospital policy procedure
13. How will you give a wound care for a dry wound?

a. Hydrogen peroxide
b.normal saline
c. 5% dextrose
d. liquid paraffin
14. On a follow-up visit after having a vaginal hysterectomy, a 32-year-old patient has a decreased
hematocrit level. Which of the following complications does this suggest?

a. Hematoma.
b. Hypovolemia.
c. Infection.
d. Pulmonary embolus (PE).
15. B-Blocker acts as anti-arrhythmic agent is?
A. isoptine
B. lidocaine
C. Norvasc
D. Tenormin
16. Patient with Hyperkalemia, which is the best way to decrease the K (potassium) level in
the blood?
A. insulin
B. lasix pumps
C. kayexalate

17. When feeding a patient with gastrostomy tube, which of the following intervention should be the
nurse's priority
a. Elevate HOB at least 30•
b. Initially flush the ostomy with 30mI water
c. Check patency by auscultation
d. heat formula in room temperature

18. Adult patient admitted the ICU, at night he became agitated, what do you expect this patient
have:
A. schizophrenia
B. depression
C. Hospital (ICU) psychosis
D. Stress or anxiety

19. Which of the following signs and symptoms usually signifies rapid expansion and impending
rupture
of an abdominal aortic aneurysm?

a. Abdominal pain.
b. Absent pedal pulses.
c. Chest pain.
d.Lower back pain.

20. 10month old communication?

a.say mama and dada


b.can smile and wave bye
bye
21. Why its contraindication to give high flow 02 to a COPD (Chronic Obstructive Pulmonary Disease)
patients?
A. because it may cause 02 toxicity.
B. to maintain breathing stimulation which initiated by the CO2
22. A patient came to ER he is expecting an
anaphylactic reaction because he ingested shellfish, the patient stated
that every time he ate shellfish it always happen, the patient have normal
vital signs, no respiratory distress, the patient have no signs of allergic
reaction what should the nurse do:
a. ignore d patient
b. give epinephrine subcutaneous
c. prepare Endotracheal set
d.insert iv line
23. A Patient on treatment of Diosmin +
Hesperidin. What Diagnosis Support this treatment.

a. GERD

b. Sinusitis

c. Diarrhea

d.Hemorrhoids
24. Which is more prone to oxygen deprivation?

a.Anemia
b. laryngeal cancer
c. rib fracture
d. GERD
25. Digoxin toxicity what is the ECG?
a. invert
ed T
b.prolonged QRS
c.chaotic P

26. pain in acute pancreatitis?


a.epigastric radiating to back
b.epigastric radiating to groin
c.LLQ radiating to groin
d.LLQ radiating to back

27. pedia with cleft palate repair what is the post op position. Nalitoakodito. SATA kase
I.prone
II. knee chest
III. left lateral
IV.sims

A. I only
B. I and II
C. III and IV
0. I,III,IV

28. the nurse says to colleage “ Mr. Hasher has no family to visit
him” a.clarification
b.emphatizing
c.sympathizing
29. What is the stimuli for respiration ?
A. Carbon Dioxide
B. Oxygen
C. Nitrous oxide

30. What is the temp of the solution of enema in children?


A. 90 o F
B. 100 o F
C. 105 o F
D. 110 o F
31. What do you expect to be ordered for depressed patients?
A. SSRI
B. ECT
C. St. John's wort

32. Patient was rushed to the emergency room due to car accident.
What would be the best to assess

a. When was the last intake of alcohol


b. What kind of alcohol was ingested?
c. what's the alcohol blood concentration
33. An infant of a diabetic mother

I.full face
II.fat
III.hair in the outer ear

A. I only
B. I and II
C. I and III
D. I, II and III

34. A nurse has a neighbor who was admitted but assigned to another nurse? What is the correct
thing to do?
a.keep it to yourself
b.get a chance to visit him
c.tell your neighbors to visit him
d. read the chart if he/she is allowed to have visitors

35. A child swallowed marble. What would indicate that the child
is having total obstruction?

a. coughing
b. gagging
c. tachypnea
36. Which would decrease lymphocytes? (select all that apply)

I. Crohns Dse
II. Lymphoma
III. Hodgkins Dse
IV. Lymphotic Luekemia

A. I and II
B. II and III
C. III and IV
D. I,II,II, and IV

37. A 3% solute, 60mg solution

a.2ml
b. 2.5
c. 5mI
d.20
Fromula: 3% means 3/100= 0.03
0.03x60= 1.8ml the close is 2mI

38. What meds are taken sublingually?

a. Lipitor
b.Nitroglycerine
c. Haldol

39. Involves absorption, distribution, metabolism, excretion of drugs

a. Pharmacodynamics
b.Pharmakokinetics
c. Pharmacology

40. ABCD of skin cancer except?


a. Asymmetry of the shape
b. borderline irregularities
c. Color not the same
d. Diameter more than 6 mm

41. Sudden infant syndrome. Verbalization of mother that needs further teaching?
a. I feel guilty my baby died.
b.I should have taken CPR training.
c. I should always check the baby while sleeping this should not happen.
d. The baby should sleep at the back.
42. Patient having seizure, what is the initial intervention of nurse?
a.Clear area from furniture
b. Put tongue depressor

43. Which would be the diagnostic result in pancreatitis? SATA


I. decrease lipase
II. Increase serum amylase
III. increase white blood cells

A.I and II
B.II and III
C.Iand III
D.I, II, III

44. A nurse admits a pt with MI to the coronary care unit. the nurse plans to do which of the ff
in delivering care to the pt.?

a. admin o2 at a rate of 2-6 L/min


b. infuse intravenous fluid at a rate of 150ml/ hr
c. begin thrombolytic therapy
d.place client in continuous cardiac monitoring

45. Room should be closed at all times. SATA


I. airborne
II. droplet
III. Contact

A. I
B. II
C. IandII
D. II and II

46. A woman diagnosed with end stage lung cancer and tells the nurse hat she is dying. Best
action of the nurse?
a.Sit with her and tell “this must be difficult time for you”.
b. Encourage the family member to stay with the patient.

47. Patient after Craniotomy have Jackson pot/hemovac. The nurse should aware that the patient is?
a.increase risk for infection
b. elevated ICP
c. Cerebral edema
48. A client who has had an abdominal aortic aneurysm repair is 1 day post op. the nurse
performs abdominal assessment and notes the absence of bowel sounds. the nurse should:

a. call the dr immediately


b remove ngt
c. feed d pt
d. document the finding and continue to assess for bowel sounds

49. patient experiencing emotional stress, appropriate therapy except:

a. guided imagery
b. Biofeedback
c. cognitive reframing
d. exercise (can't remember the complete choice but it has something to do with
exercise)

50. A nurse has a neighbor who was admitted but assigned to another nurse? What is the
correct thing to do?
a.keep it to yourself
b.get a chance to visit him
c.tell your neighbors to visit him
d. read the chart if he/she is allowed to have visitors

51. What does the nurse need to write on the assessment during 2nd day post hemi colectomy?
a. Temp.
b. Urine output
c. IVF
d.Drainage output

52. Solution use to clean pin site?


a.hydrogen peroxide
b. betadine
c. alcohol

53. Client develops bilateral wheezes, orthopnea, tachypnea, notes pitting edema +2, the nurse
suspects pulmonary edema and notifies the physician. while waiting for the dr to arrive the
nurse avoids which actions?

a. preparing to admin iv morphine


b. position client in high fowlers
c. elevate clients legs
d. prepare to admin furosemide
54. Infant with cut on the forehead, while suturing the infants hand is touching the cut. What type of
restraint?
a. belt
b. mummy
c. jacket
d.elbow

55. If the pt complain of pain when inflation of the balloon during the foley catheter insertion, the
proper nsg action is?

a. Aspirate the fluid and remove.


b.withdraw the fluid and insert more in then re inflate.
c. put lower amount of fluid inside the balloon

56. Hyperpituitarism in adult


A. Acromegaly
B. addisons
C. Cushings
D. Gigantism

57. What does it mean if the nurse signed as witness in the consent form?
a. nurse explained the procedure to patient
b. patient knows the risk of the procedure
c. the procedure was explained thoroughly to the patient

58. 72 hours a burn patient develops an infection. The nurse should place the patient under what
precaution/isolation?

A.Standard
B.Contact
C. Reverse
D.Airborne

59. Patient is experiencing a panic attack, which of the ff is the nurse's highest priority?

a. ask the client to deep breathe


b.stay with the client to reassure him
c. ask the client to breathe through a paper bag
d. administer diazepam as prescribed
60. Why is oxygen dangerous?

a. It is explosive
b. It supports burning
c. It is poisonous
d. It ignites and burns easily

61. You see in pernicious anemia patient except.

A. glositis
B. jaundice
C. paresthesia
D. Pruritus —+ IDA

62. 2 eye drops ( antibiotic and prednisone) all right exept:


A. Refrigerate the drops
B. Shake before use
C. Clean the eye
D. Give them separated 5 min apart

63. The accountability for a nurse competency is


A.Nurse
B.Hospital administration
C.Supervisor
D.Board license
64. Christmas disease is also known as:
a. thalasemia
b.hemophlia B
c. cri du cat
d.klienfelter syndrome
65. Codeine phosphate has been prescribed for patient with headache which route
A.Oral
B.lv
C.Inn
D.Rectal

66. Which of the following condition is ch.ch a dry, scaly butterfly —shaped rash across the nose and
mouth
A-Eczema
B-scleroderma
C-rheumatoid arthritis
D- Systemic lupus erythematous (SLE)
67. The stool appear when bile duct is

obstructed A.Pale
B.Yellowish
C.Gray
D.Clay
68. The person responsible for collaborating care in the surgical team is ???

A- Circulating nurse
B- Scrub nurse
C- first Assistant
D- Surgeon
69. Stomatitis what the nurse should advice the patient?
A. Take iron supplement
B. Use mouth wash
C.Increase Intake of citrus juice

70. Patient diagnosed with acute DVT attack what is indicated for the pt?
A. Ambulation
B. Stocking
C. Massage
D.Elevation

71. What the nurse should prevent the nurse student to do?
A. Deep abdominal palpation
B. Check pulse rate
C. Lung auscultation

72. BNI of an obese client is 31.how much calorie should she decrease other than her daily intake
to lose 1kg per week?

A)500
B)1000
C)2000
D)3000

Formula: 1kg=2.2Ibs
1Ib=3500caI
3500 cal x 2.2 =7700 cal
7700caI is equivalent to 1kg
bthe daily caloric intake of average woman is ranging frm 1200-2000caI/day so need to decrease
1100caI/day to lose 1kg/wk.
73.
REVIEWER – 9
1. A patient in the cardiac unit suddenly develops ventricular tachycardia on the monitor. Upon
checking, the patient has pulse and appears to be stable. Which of the following would
be anticipated by the nurse?
A.Defibrillation
B.Epinephrine
C.Lidocaine
D. Amiodarone
2. A female patient, who speaks a dialect that is hard to understand, has emergency gallbladder
surgery, during discharge preparation, which nursing action would best help this patient
understand wound care instruction?

a. Asking frequently if the patient understands the instruction


b. Asking an interpreter to replay the instructions to the patient.
c. Writing out the instructions and having a family member read them to the patient
d. Demonstrating the procedure and having the patient return the demonstration
3. A patient asks to be tested for Lyme disease. She had been bitten by a tick and removed it
herself this morning. what is the most appropriate action?
a. advise the patient to return in 4-6 weeks, when testing will yield reliable results
b. refer the patient to a physician for a precautionary administration of antibiotics
c. disinfect the skin around the bite area and instruct the patient to return if arthralgia
other concerning symptoms develop.
d. obtain a blood sample for immediate testing.
4. A patient is receiving furosemide. The nurse is aware that the action of this drug is to?
a. Act on the ascending loop of henle, excretes potassium
b. Act on the descending loop of henle ,excretes potassium, sodium and chloride

c. Act on the ascending loop of henle,excretes potassium, sodium and chloride


d. Act on the descending loop of henle ,excretes potassium
5. Which best describes Cheyne-stokes Breathing?

A. This shallow-deep-shallow pattern is followed by periods of significant apnea that


can last up to 30 seconds or longer, and then the cycle starts over.
B.A respiratory pattern characterized by periods or “clusters” of rapid respirations of near equal
depth or followed by regular periods of apnea.
C.Have a prolonged inspiratory phase followed by a prolonged expiratory phase
commonly believed to be apneic phases.
D.Completely irregular breathing pattern with irregular pauses and increasing episodes of
apnea.
6. Which of the following is the meaning of patient's autonomy?

a. Patient has the right to be informed about results and procedures.


b. The nurse respects the patient's principles of freedom, choices, self-determination
and privacy.
c. Patient has the right high quality of nursing care and international standards.
d. Patient has the right to be informed of the diagnosis and its expected signs and symptoms.
7. On finding a patient in the bathroom having a seizure, which of the following should the nurse
do first?
a. Clear the area from furniture
b. Put pillow under the head
c. Open airway with a tongue depressor
d. Record the type of movements
8. A nurse has a neighbor who was admitted but assigned to another nurse. What is the correct
thing to do?
a.keep it to yourself
b.get a chance to visit him
c.tell your neighbors to visit him
d. read the chart if he/she is allowed to have visitors
9. Which laboratory value should be closely monitored in a patient taking chloramphenicol?
a. Prothrombin time
b. Partial thromboplastine time
c. Complete blood count —+ prone to aplastic anemia ok
d. Serum potassium level
10. Which of the following would be seen in a patient with celiac disease?
A. Distended abdomen
B.Salty sweat
C.Hypopigmentation of the iris
D. Choking and cyanosis after feeding
11. Diagnostic marker for breast cancer is?
A.B2M gene
2. CD markers and T cells
3. ER
P test 4.
CA 19-9
12. As the nurse caring for the following clients, you are aware that transparent adhesive film would
be useful in which of the following patients? 1
• Indicated for use on superficial wounds such as superficial pressure ulcers, minor
burns, cuts and abrasions. Also indicated for use as a secondary dressing or to provide
catheter fixation

a. A patient with draining wound.


b. A patient with stage 3 pressure ulcer.
c. A patient with second degree burns. (superficial burns)
d. A patient with full thickness burns.
13. The nurse is caring for a 2 1/2year old male client with tetralogy of Fallot. Which assessment
findings should the nurse expect?
a. Aortic stenosis, atrial septal defect, overriding aorta, left ventricular hypertrophy
b. Pulmonary artery stenosis, intraventricular septal defect, overriding
aorta, right ventricular hypertrophy
c. Patent ductus arteriosus, Ventricular hypertrophy of left side, overriding aorta, Right
Ventricular hypertrophy.
d. Transposition of the great vessels, intraventricular septal defect, right hypertrophy,
patent ductus arteriosus
14. While teaching a patient with Bacterial Pharyngitis. The nurse is correct in differentiating between
Viral and Bacterial Pharyngitis that Bacterial Pharyngitis has?

a. Painful swallowing
b. red swollen lymph
nodes c . Exudates in the
tonsils
d. Swollen lymph nodes
15. The Researcher is using a sample of 50 workers out of 180 Employees of a department store.
The manpower is as follows

100 - Male workers


80 - Female
workers

The researcher randomly selects a group composed of


56% PlaIe
44% Female

Which of the following sampling types is being implied?


a. Snow ball
b. Stratified
c. Quota
d. Quasi experimental
16. A client with type 2 diabetes is planning to go to the gym to exercise high intensity running for
30 minutes. Which of the following advices about calorie intake should be given to the patient
prior to exercise?
a. 60 calories of carbohydrates
b. 120 calories of carbohydrates
c. 300 calories of carbohydrates
d. 500 calories of carbohydrates
17. You are caring for a client with end stage renal disease. Which of the following would you
anticipate to be seen in a patient that led to differentiate other renal disease manifestations with
end stage renal disease?
a. Anemia
b. Increase serum creatinine
c. Elevated blood urea nitrogen
d. Hyperkalemia
18. 1st choice for feeding a patient with dysphagia and stroke:
A)NG tube
B)PEG
C)TPN
19. The nurse is caring for a client with hypertension. The nurse understands what about excess fluid
volume

a. It increases cardiac output.


b.It increases peripheral dilation,
c. It increases blood flow to the kidneys.
d. It increases serum electrolytes.
20. Hormone highest during 7th month of pregnancy?

A) progesterone
B)estroge
n C)hCG
21. Which of the following vitamins is given for elderly because they have poor ability to store them

a. A
b. D
c. C
d. all
22. The nurse enters the room of a patient who is in the clonic phase of a tonic clonic seizure.
The initial nursing action should be to:
A. Insert a padded mouth gag
B.place some padding under the head
C.Restrain the patient
D. Obtain equipment for orotracheal suctioning
23. Pain behind eyes
a. Migraine
b. Cluster headache
c. Tension headache
24. Bladder cancer risk factors
a. Ulcerative colitis
b. Long term catheterization
c. Smoking
d. all
25. Nsg diagnosis priority for pt with renal calculi:

A)fluid volume deficit


B) pain
C) risk for bleeding
D)risk for oliguria
26. You are taking care of Mrs.L 66 yrs.old who is terminally ill with ovarian cancer stage IV.When

caring for a dying pt. you will perform which f the following activities?
a.Encourage the client to reach optimal health
b.assist the client to perform activities of daily living
c.Assist the client towards a peaceful death
d. Motivate the client to gain independence
27. Which anemia occurs in vegetarian client?

A.Cooleys Anemia
B. IDA
C.Pernicious Anemia
D. Aplastic Anemia
28. A client is admitted with a diagnosis of hepatitis B. In reviewing the initial laboratory results,

the nurse would expect to find elevation in which of the following values?
A) Blood urea nitrogen
B) Acid phosphatase
C) Bilirubin
D) Sedimentation rate
29. Physician prescribed which drug for a client who displays increased psychomotor activity....

A:zlofit
B:HaIdoI
C:MarpIan
D:ChIoraI Hydrate
30. In the third stage of labor,

what are the nursing interventions?

A.Mild fundal push

B.May occur up to 3 hours after infant delivery, wait patiently

C.Administer oxytocin, ZV or III as bolus

D. Ask patien to bear down slightly


31. Patient have delusion of persecution stating that CIA are after him. What is the best response
of the nurse?

A. "What makes you think that CIA are after you?"


B. "I understand your fear but I'm afraid its not true."
C. "That a lie. I dont see CIA here."
32. Positive coombs test is:
A) mother Rhnegative and baby Rh+ve
B) mother Rh+ve and baby Rhnegative
C) mother Rh+ve and baby Rh+ve
33. Nurse evaluates which client is at risk for hypernatremia:?
A)50yr old with pneumonia,diaphoresis,high fevers
B) 62yr with CHF takng loop diuretics
C) 39yr old with diarrhea and vomiting
D) 60yr old with lung cancer and SIADH
34. Pregnant woman complain legcramps during the night, the nurse instruct her to:

A)elevate leg above heart level


B)dorsiflex the foot
C)assume prone position
D)hot compress over affected leg
35. Which situation should have standard precaution other than using gloves?

a. Barium enema administration


b. Interstitial radiation on oral cancer
c. Renal arteriogram, intravenous pyelogram
36. which direction must be given to an adolescent being treated with tetracyclne for acne

a.eliminate fatty foods from daily diet


b.wash afected areas thouroughly
c.keep out direct sunlight
d.use only oil based skin care products
37. Patient is diagnosed of UTI, which is the best way to assess temperature?

A.Oral
B.Tympanic
C.AxiIIary
D.RecataI
38. The nurse would expect the cystic fibrosis client to receive supplemental pancreatic enzymes
along with a diet

A) High in carbohydrates and proteins


B) Low in carbohydrates and proteins
C) High in carbohydrates, low in proteins
D) Low in carbohydrates, high in proteins
39. Which of the following nursing assessments in an infant is most valuable in identifying serious

visual defects?
A) Red reflex test
B) VisuaI acuity
C) Pupil response to light
D) Cover test
40. How would you know if child who swallowed marble has complete obstruction of airway?

A.Inability to talk

B. Gagging

C.Coughing

D. Tachypnea
41. Dry fire extinguisher:
A) paper
B) electric
C) flammable liquids
42. A client with peptic ulcer is being assessed by the nurse for gastrointestinal perforation. The
nurse should monitor for:
a. (+) guaiac stool test
b. Slow, strong pulse
c. Sudden, severe abdominal pain
d. Increased bowel sounds
43. Mouza who was diagnosed with brain tumor was scheduled for craniotomy. In preventing the

development of cerebral edema after surgery, the nurse should expect the use of:
a. Diuretics
b. Antihypertensive
c. Steroids
d. Anticonvulsants
44. The nurse is aware the early indicator of hypoxia in the unconscious client is:

a. Cyanosis
b. Increased respirations
c. Hypertension
d. Restlessness
45. A female client is receiving IV I \annitoI. An assessment specific to safe administration of the said

drug is:
a. Vital signs q4h
b. Weighing daily
c. Urine output hourly
d. Level of consciousness q4h
46. While performing a physical assessment of a male client with gout of the great toe, Nurse

Vivian should assess for additional tophi (urate deposits) on the:


a. Buttocks
b. Ears
c. Face
d. Abdomen
47. Type of headache that has eye pain that radiates to temple?

A.Migraine

B. Cluster
C.Tension
D. Sinus
48. Nasal cannula 02 concentration should NOT exceed

a. 2%
b. 4*/a
c. 8%
d. 24%
nasal cannula (1-6L) 02
concentration IL - 24 %
2L- 28%
3L - 32%
4L - 36%
5L - 40%
6L- 44%

formula: FiO2 room air (20%) + 4% per liter of oxygen flow


NOTE: this only applies for NASAL cannula
49. Which should not be given in
pt having diarrhea?

A.rice

B.toast

C.plain crackers

D. diluted juice
50. Which of the following regarding nursing diagnosis:
A)medical pathology
B)treatment
C)actual problem
D)lab result
51. A female client is receiving IV Mannitol. An assessment specific to safe administration of the said
drug is:
a. Vital signs q4h
b. Weighing daily
c. Urine output hourly
d. Level of consciousness q4h
REVIEWER – 10
August 28, 2013 exam questions. (this is the best I could squeeze out of my brain, hope this helps.
the choices in bold are my answers, NOT SURE WITH MY ANSWERS)

1) The physician ordered pain medication STAT. Which medication order should the nurse
question?
a. prophoxyohene
b. methadone
c. Meperidine
d. Tramadol

2) Common breathing pattern in DKA patients


a. Deep rapid labored breathing
b. Slow shallow breathing
”kussmaul breathing

3) In administering eyedrops, nurse is aware that OS means

a. left eye
b. right eye
C. both eyes

”OD- right eye ”OU- both eyes


4) In a patient post-cataract surgery, nurse should instruct him to
a. Avoid lifting objects more than 7kg
b. Avoid climbing stairs
C. Complete bed rest with bathroom privileges
d. Avoid watching television
”may increase pressure in the eye?

5) A mother came to the clinic with her 12month old child, his birth wt. is 3.2 kg, by now the
nurse knows that the child should weigh approximately
a. 4kg
b. 6kg
C. 8kg
d. 10kg
”wt doubles by 6mos and triples by 12mos.
6) A mother asked the nurse if her 5 year-old daughter can attend a birthday party after
diagnosed with mumps Sdays ago. The nurse's most appropriate response would be:
a. She can attend the party as long as the other children in the party are immunized already
b. She can attend the party
c. She can't attend the party because she can still spread the virus
”mumps is still contagious by Sdays

7) Which is not part of Tetralogy of Fallot


a. Hypertrophy of right ventricle
b. Ventricular septal defect
c. Transposition of the great vessels
d. Pulmonary stenosis
”PORV —pulmonary stenosis, overriding of aorta, right ventricular hypertrophy, ventricular septal
defect
8) Eye pain, runny nose, watery eye. What type of headache manifest this symptoms?
a. Migraine
b. Tension headache
c. Cluster headache
d. Sinus headache

9) What causes irritability in a patient with cast


a. Pain, immobility, dietary restriction
b. Immobility, edema, itchiness
C. Pain, edema, inability to do adls
d. Fluid restriction, pain, immobility

10) RDA of calcium


a. 300
b. 500
c. 1500
d. 3000
Nomale Serum calium 8.5- 10.5 mg/dl

11) High in saturated

fats I-beef
II-nuts
III-olive oil
a. I only
b. I, II
c. I, III
d. I, II, III
”olive oil- unsaturated fat, specifically monounsaturated
12) Obese client with bedsores, what is the priority nursing diagnosis
”I encountered this question twice in my exam, not sure with the answer though.
a. ineffective health maintenance

b. imbalanced nutrition: more than body requirements


c. risk for infection

d. impaired skin integrity

13) What is not elevated in COPD‘


a. 02
b. CO2

14)When feeding a patient with gastrostomy tube, which of the following intervention should be
the nurse's priority
a. Elevate HOB at least 30°
b. Initially flush the ostomy with 30mI water
c. Check patency by auscultation
d. heat formula in room temperature

15) Which is considered as a primary prevention?


a. Health teaching regarding insulin injection in a type 2 diabetic patient
b. Recommending regular exams and screening tests for a diabetic patient
c. Immunization against infectious diseases
d. Stroke Rehabilitation program

16) Which of the following gases is responsible for initiating respiratory drive, stimulus
for breathing?
a. Oxygen
b. carbon monoxide
c. Carbon dioxide
d. Nitrous oxide
17) In cranial nerve I, what is expected to be assessed?
a. Eye movement
b. Vision
c. Smell (let patient smell coffee, etc.. was given in the choices)
d. Taste

18) When doing bed bath for a child, the water temperature should be
a. 110-115°F
b. 100-105•F
c. 115-120°F
d. 90-95°F
”adult- 110-115 (source:mosby’s)

19) In dealing with a severely depressed client what is essential with regards to
therapeutic communication
a. The process is focused on the patient
b. Nurse-patient process recording
c. Show sympathy
d. Divulge nurse's personal info to gain the client's trust

20)A dry powder extinguisher can be used for the following:

I- wood, paper, cloth


II-flammable liquids

III-Cooking oil
IV- Electrical Equipment
A. I only
B. I, II, III
C. I, II, IV
D. I, II, III, IV

”a foam or a carbon dioxide type of extinguisher can be used with cooking oils and fats

21)Initial manifestation of complete airway blockage in a 3 year old boy


a. Gagging
b. Coughing
c. Inability to talk
d. tachypnea
22) The nurse enters the room of a patient who is having seizure inside the bathroom. The
initial nursing action should be:
a. Insert a padded tongue depressor
b. Put a pillow under the head
c. Restrain the patient
d. Clear the area with furniture

23)It involves absorption, distribution, and excretion of a certain drug


a. Pharmacodynamics
b. Pharmacology
c. Pharmacokinetic

24)Purpose of giving 2 Ipm 02 in COPD patient


a. Prevent 02 toxicity
b. Maintain hypoxic drive

25) Why is oxygen dangerous?


a. It is explosive
b. It supports burning
c. It is poisonous
d. It ignites and burns easily

26)Nursing responsibility in the 3rd stage of delivery, especially in the expulsion of placenta

A. Mild fundal push


B. May occur up to 3
hours after infant delivery,
wait patiently
C. Administer oxytocin, IV
or IM as bolus
D. Ask pt to bear down slightly
27)Hypochromic, microcytic rbc is seen in patients with
a. Folic acid deficiency anemia
b. Iron deficiency anemia
c. Aplastic anemia
d. Pernicious anemia

28)Post op cataract surgery, avoid vomiting because


a. May cause infection
b. Increases IOP
29)Patient with diarrhea, don't give:
a. Rice
b. Cracker
c. Dry toast
d. Diluted fruit juice
30) Medication for patient with ventricular tachycardia who is hemodynamically stable
a. Atenolol
b. Verapamil
c. Amiodarone
d. Lidocaine
Ventricular Tachycardia- is defined as three or more PVCs in a row, occurring at a rate
exceeding 100 beats per minute

31) Meds given sublingually

a. nitroglycerin

b. aspirin

c.tramadol

32) Chest pain worsens when drinking cold drinks —angina pectoris

33) vitamins given for elderly because they have poor ability to store them

a. A

b. D

c. C

d.all

34) lmmediate intervention for unconscious patient after vehicular accident.

a. Jaw thrust
b. Oxygen
c. Assess LOC

35)BIadder cancer risk factors

I- Ulcerative colitis
II- Long term

catheterization III-Smoking

a. I, II

b. II, III

c. I, III

d.I, II, III

36) Position for obese pregnant who is restless and cannot sleep at night — dorsal recumbent position

37) 250mg, pt. bsa is .54, adult bsa 1.73

My answer is 78, but it is not given in the choices. Go with the nearest answer which is 75

.54/1.73 x 250= 78

38) infant becomes jittery after delivery from a diabetic mother

a.hypoglycemia, hypocalcemia

b.hypoglycemia, hypercalcemia

39) medication for a depressed patient

-SSRI

40) patient experiencing emotional stress, appropriate therapy except:

a. guided imagery
b. Biofeedback
c. cognitive reframing
d. exercise (can't remember the complete choice but it has something to do with exercise)

41) Nurse is interviewing a psychiatric patient, when patient was asked about her parents her
response was “I love dancing, love the color blue...etc”
a. clang association
b. looseness of association
c. word salad
d.depersonalization

42) Patient on PCA, complains that pain does not go away. Nurse's most appropriate response:
a. check the machine
b. assess patient further, check vital signs
c. tell patient that this is normal, and pain will subside soon
d.administer another dose of pain medication

43) essential component of the therapeutic relationship


a. consistency
b. empathy
c. rapport
d.sympathy

44) patient is experiencing a panic attack, which of the ff is the nurse's highest priority?
a. ask the client to deep breathe
b.stay with the client to reassure him
c. ask the client to breathe through a paper bag
d.administer diazepam as prescribed

45) a possible cause of basal cell carcinoma


a. oily hair and dandruff
b.baldness
c. daily washing of hair with shampoo
d. contact with arsenic

46) patient have delusion of persecution. Stated that someone is after him. What is the best
response of the nurse?
a.what makes you think that someone is after you?
b.that's a lie, there's no one here
c. I understand your fear, but I'm afraid it's not true

47) cleft palate predisposing factors


Mother who took antiseizure meds during pregnancy

48) commonly affected body system in clients with electrolyte imbalance


a. cardiovascular
b.renal
c. endocrine
d. neuromuscular

49) which situation should have standard precaution other than using gloves?
a. barium enema administration
b.interstitial radiation on oral cancer
c. renal arteriogram
d. vomiting patient on brachytherapy
50) patient diagnosed with osteoporosis, what should be recommended?
a. bed rest with bathroom privileges
b.keep affected part immobile
c. weight bearing exercises

51) jaundice during the first 24 hrs of the infant's life


a. physiologic jaundice
b.ABO incompatibility

”pathologic jaundice was not included in the choices

52) A diabetic patient eats sweet food brought by his wife. nurse should?
a. Reassess the patient and wife behavior and know they are eager to modify lifestyle
b. Reinforce the importance of diet in a persuasive manner
c. Tell the dietician to inform them again about the diet because he is more knowledgeable.

53) Urine collection, what should the nurse consider?


A. Sterile container
B. Menstruating women may not access accurate result
C.Strict aseptic technique
D. Time of collection

54) The following is true about "RACE" when on fire, EXCEPT?

A. Alert everyone in the building where a fire is and help them out
B. Have someone activate the alarm and fire extinguisher
C. As the last one goes out the building, ask him to close all doors/window
D. After everyone had evacuated the building, have a head to head count to know if someone is missing

55)Which of the following are seen in a newborn with gestational diabetic mother?
I. Newborn has full face
II. Newborn have excess fats
III. Newborn have excessive growth of hair behind ears

A. I & II
B. I & III
C. II & III
D. I, II & III

56) 72 hours a burn patient develops an infection. The nurse should place the patient under
what precaution/isolation?
A.Standard
B.Contact
C. Reverse
D.Airborne

57) 4 days post-op abdominal surgery of an obese patient coughs and vomits. The patient is at risk for?
A. Pneumonia
B. Adhesions
C. Dehiscence- disruption of surgical incision or wound. Evisceration- protrusion of wound
contents
D. Paralytic ileus

58) Patient undergone total knee arthroplasty. Nursing assistant reported to RN that the patient's
leg is red and warm to touch. Patient complains of leg cramps. What should the RN do?

A. Tell the nursing assistant that this is expected as normal


B. Check when last pain medication was given
C. Notify the physician
D. Tell the nursing assistant to massage the leg

59) 3% solution- with 60mg/ml as stock. How much to give?


Dnt know if I got this right. This is how I computed it .03x60= 1.8
Closest to the choices is 2. That's what I chose.

60) Signs and symptoms of perforated ulcer?

A. Palpable mass over the abdomen


B. Tender, rigid abdomen
C.Frank blood in the stool
D. Biled colored vomitus

61) Which is more prone to oxygen deprivation?

a. Anemia
b. laryngeal cancer
c. rib fracture
d. GERD

62) if the pt complain of pain when inflation of the balloon during the foley catheter insertion, the
proper nsg action is?
a. Aspirate the fluid and remove.
b. withdraw the fluid and insert more in then re inflate.
c. put lower amount of fluid inside the balloon
p. Attach the syringe to the balloon port of the catheter. Inject the water slowly to inflate the balloon. If
the water will not inject easily or the patient complains of pain, deflate the balloon completely and
advance the catheter further, then re-inflate.
63) Asepsis means?

A. No microorganisms and spores


B. No infectious microorganisms

64) Temperature of Enema for child


a. 95 F
B. 100 F
C. 105 F
D. 110 F

65) Which help nurses in prioritizingplan of care?

A. Nursing care plan


B.doctor's orders
C. Nurse's progress notes from previous shift
D. kardex

66)Pt recently experienced break upin marriage, socially withdrawn, not eating. How would you plan
for his adequate nutrition?

A. provide high caloric, highprotein diet


B. Include family in planning diet
C. Give all the food he desire

67)How to assess if infant isreceiving enough milk -4Weigh diapers

68) As a junior nurse you've heard your senior nurse and his visitor talking about your patient.
Your appropriate action should be:
a. interrupt the conversation right away
b. ask them to talk somewhere else
c. don't interrupt the conversation. Talk to your senior nurse privately
after their conversation

69) which is true about HIV/AIDS?

A. It takes 1 yr to
develop aids after HIV
infection
B. Anyone can have HIV /aids
C. Men have the most increasingpopulation to have HIV /aids
D. Elderly people cannot have HIV /aids

70) Child is obese, parents areobese as well. On initial assessment of the child what is most likely the
cause?
A. High intake of fats
B. Genetic disposition
C. Watching too much TV
D. Lack of exercise

71)in a client with damaged cerebellum expect to have problems with BALANCE and
COORDINATION

72) The nurse is caring for a client with a history of falls. The first priority when carinp for a
client at risk for falls is;

A. placing the call light for easy access


B. keeping the bed in the lowest possible position
C. instructing the client not to get out of bed without assistance
D. keeping the bedpan available so that the client does not have to get out bed

73)before administering ifliximab, it is important to


assess a.wbc
b. liver function
c. tuberculin test

74)CVA with Left sided weakness — pillow between legs

75) patient with CGN, previously had sore throat, the possible cause is

a.Group A Beta Haemolytic Streptococcus

b.streptococcus pneumonia

76) Protrusion of the nerve roots and spinal cord

a. Meningocele
b. Myelomeningocele
c. Spina bifida
d. Spina bifida occulta

77) what causes the closure of foramen ovale

- pulmonary pressure decreases and the leh atrial pressure exceeds that of the right

*the other choice mentioned something about the umbilical cord. It's the choice above that I chose.
78) Check the legs, lower extremities — what type of anemia

a. Thalassemia
b. Aplastic
c. Sickle cell

79) Stool exam for patient with melena

a.Guaiac test
b.lower GI series
c. colonoscopy

80) child with ADHD, showed inappropriate behavior on socializing with other kids

a. isolate patient for inappropriate behavior

b. let him explore more so that he’ll realize his fault???

81) Female catheterization — dorsal recumbent position

82) mother asked the nurse what the doctor is doing because her baby extended her arms
after the baby is thrown in the air. The nurse is aware that this kind of assessment is done to
check the:

a.moro reflex

b.rooting reflex

c. Babinski

83) 35% burn patient experiencing tachycardia, tachypnea. Nurse is aware that the possible cause of this
iS:

4 hypovolemia

84) side effects of niacin extended release tablet, except:

a. rashes

b.hot flushes and headache

c. back pain

d. gout
85) The nurse is caring for a client with a history of falls. The first priority when caring for a client
at risk for falls is;

A. placing the call light for easy access


B. keeping the bed in the lowest possible
position C.instructing the client not to get out of bed without
assistance
D.keeping the bedpan available so that the client does not have to get out bed
86)Common sign of meningitis in newborn:
A)nuchal rigidity
B)brudzinski sign
C)restlessness and vigorous crying

87) The nurse should know that the normal therapeutic level of lithium is :

a..6 to 1.2 meq/L


b. 6 to 12 meq/L
c. .6 to .12 cc/ml
d..6 to .12 cc3/L

88) the nurse checks for residual vol before giving a bolus tube feding with a ngt and obtains a
residual amnt of 150 ml. wat is the app axn?

a. hold feeding
b. reinstill and continue
c. elevate head and admnster feeding
d. discard d residual amnt then feed
REVIEWER – 11
AUGUST 15, 2013 exam (my friends exam)

1. In Pt with increased osmotic pressure, what solution would you give to decrease OP?
a. Hypertonic
b. Hypotonic
c. Isotonic
d. Colonic
2. What will you give to pt with paracetamol overdose?Acetylcysteine
3. What is quasi experiment?
4. Delayed closure of the anterior fontanel may be caused by? - pulmonary pressure
decreases and the left atrial pressure exceeds that of the right
5. In rheumatoid arthritis, assessment on tarsals area is to confirm level of?
a. Uric acid
b. Creatinine
c. Drug effectivity
d. BUN
6. What is the BEST way to assess the proper breast feeding technique?
a. The position of the baby
b. Weigh the dia per daily
c. The breast is not tender
d. The fontanel is not sunken
7. In 3% solution, the order is Smg how much will you give to dilute?— ( I dunno the answer)
8. What is true about HIV/AIDS?
a. HIV exposure of 1 year may cause AIDS
b. HIV/AIDS is different
c. HIV/AIDS does not affect on older adult
9. In infant 12mos oId,came to the clinic for ff up checkup what is the possible wt gain?
A.double,
B.triple
C.quadruple
10. Which of the ff is the initial sign of meningitis? >restlessness /LOC signs and symptoms
11. Which of the ff discharge teachings to include in pt with cataract to prevent increase of IOP?
a. Avoid ambulation, with comfort room previliges
b. Work with lifting up to 7 kg
c. Daily exercise
d. Drug instructions
12. Which of the ff CHD sx that can be seen in Tetralogy of fallot?
a. Polmunary stenosis
b. Ventricular septal defect
c. Left ventricular enlargement
d. Transposition of great arteries
13. If the Practical nurse takes vital sign and the registered nurse gives medication what
Models of Nursing Care Delivery they use?
A. Primary Nursing
B. Holistic Nursing (aka. total patient care)
C. Team Nursing
D. Functional Nursing

14. Patient allergic to penicillin what other drugs to avoid?Cephalosporin


nd
15. Normal weight gain during 2 trimester? 1lbs/week or 4.1kg in whole trimester.

STUDY THE FOLLOWING TOPICS


• Anemia
• Types of Headache
• Normal values Potassium and Sodium
• Leukemia
• Meningitis
• Menierse disease
• Gestational Dm
• Insulin
• Hypovolemic
• V'tach drugs
• IV calculations
• Evidence based nursing
REVIEWER – 12
120 HAAD exam question

You have to do your part in finding the answers for some

questions. 1- patient is on digoxin. What is the drug of choice?


- Lasix

2- post operation patient always asking for analgesic (over seeking). What is the most
appropriate nursing intervention?
- inform the physician to put the patient on regular analgesic
- tell the patient that it's a fake feeling
- Increase patients analgesic dose

3- patient with Digoxin with Hyperkalemia, what do you expect the ECG rythem
- peaked,?? (check)

”F/af P waves, Prolonged Pr inferva/, wicfenerf QRS comp/ex, Tall peakerf T wave

4- a woman with dysmennorhea, how can the RN know that she is pregnant without
any investigations?

5- A patient with diabetic foot, during the discharge plan, how can the nurse know
that the patient understands the correct way to take care of his feet?
- I'll check my foot every clay {inspect}

6- when foleys is inserted, how does it fixed?


- inflation of the balloon.
- rotate the cathter and fix it by tape.

7- patient with acute renal failure, after investigation (Blood and urine) what do you
expect to have?
- creatinine is high.

8- how can you assess the severity of CVA (Cerebrovascular Accident)


- the affected area in the brain
- block of the artery
- Nerves affected

9- What the suitable position for CVA patient, during doing oral cavity care.
- Supine
- lateral
- prone
10- During NGT (Nasogastric Tube) insertion, the nurse noticed a resistance, what is
the suitable Nursing intervention?
- remove the NGT.
- apply more power
- Rotate the tube

11- During NGT insertion the patient become cyanosed, Nsg intervention?
- remove the NG and monitor.
- Give 02.

12- During NG feeding, why it suppose to be slowly feeding (by gravity)?


- because the patient may develop Diarrhea
- because may develop abdominal destention.

13- what is the ideal way when you make suctioning to a patient
on Mechanical Ventilator?
- Hypervenf//af/on (by Ampobag) pre and post suctioning.

14- How the RN assess that the Chest tube s are working proberly?
- fluctuation (oxalating)

15- How to assess an emphysema with palpitation?


- When crack/es sensation tznz/er fhe skin is be/I {pa/pafec/}

16- the most common risk factors of developing a pneumonia?


- pts on /I/fechan/ca/ Ventilator.

17- Pneumonic Patient , has purulent mucous, how the nurse can assist the excretion of
this mucous?
- by percussion.

18- patient is planned for discharge on diuretics, how the nurse can know
the patient understood the care plan ?
- “will measure and document the intake/ output”
- “ I'll weigh my self daily”

19- Renal Failure patient for discharge, health education??


- avoid food with high K (potassium), Banana,etc

20- Patient with Hyperkalemia, which is the best way to decrease the K
(potassium) level in the blood?
- insulin, lasix pumps
- kay oxalate
21- the Description of good granulation tissue formation?
- pink, soft and may b/eecf when being toochecf

22- patient on diuretic, what the RN must keep in mind to monitor.


- Pulse.
- Potassium level.
- Blood Pressure.

23- Patient with GI (Gastrointestinal) (GI Bleeding), stool color?


- Dark (Upper GI B/eei:fingj, (Bright Lower GI B.) + bed odor (Melena)
24- the purpose of let the patient with esophagus Varices having cold water ?
- cold water makes V'asoconstrfcfion, prevent bleeding.

25- the Evidence that the patient may have Anorexia nervosa?
- Anemia

26- During Dealing with a Geriatric Patient , what the nurse should expect?
- difficulty swallowing
- Speaking slowly

27- .patient with CVA, how the nurse can assist to enhance the facial movement?
- encotzrage chewing and smiling.

28- patient with an amputated leg above the knee, complaing of pain in the
his amputated knee, what is the appropriate Nsg intervention?
- tell the pt that this a fake feeling.
- “I understand what you feel, bla bla. The nurse have to realize the fantom Pain).

29- post op patient had a thyroidectomy, how can the nurse realize that the
pt developed a parathyroid injury?
- misc/e twitching.

30- the most dangerous arrhythmia?


- V-tach (Ventricular tachycardia.
- VF (Ventricular fibrillation)
- braycaria

31- a pediatric patient with VSD (Ventricular-Septal Defect), the nurse must know that
this disease is?
- Cyanotic r/isease.
- may or may not need surgical repair.
32- during assessing the understanding of health education for a patient about elastic
stocking, the patientstates?
- ” I will wear them during fhe day, and fake them o/'be/'ore s/eep/ng”.

33- the most common risk factor after thigh open fracture injury is?
- Pulmonary empoIism.(fat embolism)
- Bleeding.
- Severe pain.

34- ICP (IntraCranial pressure) normal value is?


- 10-20 cm h2o.

35- how is the appropriate nursing care for a diabetic (DM) patient's nails?
- cut straight, then file.
36- Health Education for a diabetic patient, before having a bath the patient must
measure the water temperature by?
- put his e/6ow in the water.
- use a thermometer.

37- Physician order “give 10 IU mixtard (mixed) with 5 IU actrapid (clear) insulin........) ,
the nurse should?
- withdraw actrapid then Mixtard.
- withdraw mix then actrapid.

38- During medication preparation, the nurse noticed unclear label, or unclear expiary
date of a medication, what the appropriate nsg intervention?
- return fo fhe pharmacy fo 6e rep/acerf.

39- When a nurse write an incident report about an error he/she does, it is an example

- confidentiality
- accountability

40- when the RN delegates a PN to do a procedure, in case of any mistakes who will
be responsible?
- RN
- PN
- Supervisor
- Physician.

41- Patient on Warfarin (Anti coagulation), how the nurse know that the pt understood
his health education, all are correct expcept?
- I will shave by raser instead of shaving set.
- I check (inspect) my body daily of bruises.
- Continuously lab check especially INR level.
- its normal to have dark urine

42- usually pts on warfarin, they must regularly check..


- bleeding time
- INR or PT
- ESR (Estimated sedemintation rate).
- PTT

43— usually pts on Heparin, the nurse must regularly check..


- bleeding time
- INRor PT
- ESR (Estimated sedemintation rate).
- PTT?

44- Bed ridden patients hoe have low weight (slim), with poor nutrition, immobilized, are
at high risk to develop..
- Bed Sores
- DVT (Deep Vein Thrimbosis)

45- when changing the position for a patient with skin traction (with fractured leg), the
appropriate nsg intervention?
- Hold the weighf {the traction) before changing the position.

46- the protective infection precaution equipment when dealing with a meningitis case
is?
- surgical face mask (droplet)
- Gloves.

47- to have the best effectiveness when using a skin traction is?
- free hanging.

48- when the nurse deals with a psycho patient with severe depression, the nurse
needs toilet, the appropriate nsg intervention is?
- tell the patient that he will come back in 5 minutes, and instruct him not to move
until he come.
- make any other nurse to cover (replacement).

49- in an Acute Bacterial Meningitis, the CSF (CerebriSpinal Fluid) investigation will be:
- low glucose level.
- high glucose level
- high protein level.
- low protein level
50- in PACU (Post Anesthesia care Unit), the nurse priority during monitoring the pt is?
- Blood pressure (BP)
(in case you have an airways and o2 saturation in the choices not the BP that will be the
correct answer)

51- the drug of choice for bradycardia


- Atropine.
- Digoxin.
- epinephrine (Adrenaline)
- norepinephrine.

52- for terminal stages pts who complaining of pain, asking (Morphine)
- give when they complain pain.

53- the best position during having a kidney biopsy is?


- Prone with sand bag support behind the Rt- Lt abdominal area.
- lateral

54- the most complication may the patient have after the liver biopsy procedure is?
- severe Pain.
- B/eec/ing (Bile)

55- Nsg intervention for an amputated leg with a biological patch is?
- Elevation above pi//ow — to prevent contractures.

56- severe dehydrated baby, which of the following the nurse must expect as a sign:
- crying without tears.

57- Apgar score:


- 0-3 severe distress
- 4-6 Need observation
- 7-10 No problem

57- In Renal calculi case, urine analysis will appear:


- high WBC (white Blood Cells)
- High creatinine.
- high RBC (Red Blood cells)

58- when you are speaking (communicating) to a CVA patient:


- give the patient enough time to speak (because he/she speaking moving slowly)
- Encourage the patient to speak faster.
- act as you understand what he was speaking then ignore.
59- A patient with high ICP (Intracranial Pressure), What do you expect the patient to
develop:
- coma
- Seizure
- Blindness

60- How to assess the pediatric tissue perfusion/ Breathing


- Capillary refi// fo ge < 2 seconi:Is.

61- a patient who recently lost his mother, after being informed he said “No she is
coming today to visit me”, this patient considered in which stage of grieving process?
- Acceptance.
- Oenia/
- Depression
- Stress

62- Before giving Digoxin, what Must the nurse do?


- Assess the BP
- Assess the RR
- Assess he HR
- assess the O2 saturation

63- signs of Bipolar:


- hyperactivity

64- Health Education for a patient who had total Knee replacement?
- not fo cross fhe legs

65- First choice for feeding a patient with Dysphagia and stroke:
- NG take.
- PEG
- TPN

66- Heavy smoker are at high risk to have:


- Hypertension
- CAD (Coronary Artery Diseases)
- stroke (CVA)

67- which of the following considered as (Plasma Expander)?


- Mannitol
- RBCS
- Allsumin
- Perfalgan
68- why its contraindication to give high flow O2 to a COPD (Chronic
Obstructive Pulmonary Disease) patients?
- because it may cause O2 toxicity.
- to maintain breathing stimulation which initiated by the CO2

69- Picc line , when be used for the first time, what you expect from the physician to do?
- withdraw to check if you have food blood flow before using.
- CXR (Chest X-Ray)
- good and firm dressing.

70- which of the following is correct regarding Chest drainage system Discontinue?
- slowly remove the tube — suture- dressing
- clamp- instruct of inhalation then hold on- remove — tie the wound- dressing

71- post Bronchoscopy patient, the nurse should observe before starting feeding:
- Gag ref/ex
- wait bowel movement
- NPO (Nothing Per Oss) for 6 hrs then feed.

72- to irrigate a colostomy stoma, the nurse should use:


- Tepid water
- normal Saline
- Ringer lactate
- Distilled water

73- Nursing diagnosis as priority for a patient with Renal calcholie:


- Fluid volume deficit
- Pain
- risk for bleeding
- risk for oligurea

74- what should the nurse advice a Dm patient regarding insulin use?
- Small meal — Exercise- insulin
- insulin — sleep- exercise
- sleep- exercise — insulin

75- a patient with pancreatitis clinical investigation markers are all except:
-Amailaise
- Lipase
- low serum Ca level
- high serum glucose level
- hypernatremia

76- B-Blocker acts as anti arrhythmic agent is?


- isoptine
- lidocain
- Norvasc
- Tenormin

77- signs of duodenal ulcer:


- continuous pain
- intermittent pain.
- pain relieved by meals
- pain increased by meals

78- one of the following is correct regarding Dehydration signs (pediatric)


- high HR
- low skin turgor
- crying with no tears

79- Adult patient admitted the ICU, at night he became agitated, what do you expect
this patient have:
- schizophrenia
- depression
- Hospital (ICU) psychosis
- Stress or anxiety

80- post laparatomy patient, your advice when he wants to cough is:
- to support fhe abdomen by his hand be/'ore coughing

81- with pre-exlampsia , the nurse expect: (check the textbook)


- high Na (hypernatremia), low K (Hypokalemia)?

82- Nsg diagnosis for a patient with Gestationl DM? (check the textbook)
- CVA
- Low BP
- Placenta Previa
- Poly Hydro minus

83- Type of Anemia, why..? (check the textbook)


- Low folic acid

84- DM insepidus, with old patient , you expect : (check the textbook)
- Hyponatremia
- Hypoglycemia
- high crealtinine — urine analysis
85- Most Priority Nsg action post “ Electroconvulsion Therapy” is?
- Put the pt on lateral position
- change position every 15 min
- ask how doe the pt feel.

86- When the RN prepare a dose of 75mg of pethidine, what must the nure do with
the residual amount in the 100 mg pethidine ampule?
- Discard if

87- Nursing meaning for the pts principle of Autonomy?


- pt has the right to be informed about results and procedures.
- the nurse respects the patients principles of freedom, choices, self determination and
privacy.
- pt has the right for high quality of nsg care and international standards.

88-Effectiveness of 02 therapy for a pt with COPD ?


- HB
- PH and 02 sat
- CBC, ABGs, O2 Sat.
89- with duretics administration, the nurse must be aware of:
- high BP
- weak pulse
- muscle twitching

90- first priority Nsg interventions purpose with Alzhaimer pts is:
- to cure the disease
- giving medicaton to minimize fhe Signs and symptoms of Alzhaimer.

91- first priority when dealing with unconscious traumatic pt received in the ER?
- jaw thrust maneuver.
- maintain airways and breathing and 02 therapy
- assess level of consciousness.

92- Rectal tube insertion procedure, all of the following steps are correct except:
- Lubricate the rectal tube.
- insert 4-6 inches
- assess for abdominal distention before and after insertion.
- leave the tube for 40 minutes.

93- if the pt complains of pain when inflation of the balloon during the foleys catheter
insertion procedure, the proper nsg action is?
- Aspirate fhe fluid and remov'e.
- withdraw the fluid and insert more in then re inflate.
- put lower amount of fluid inside the balloon
94- Diagnosis markers of thalassemia? (check the textbook)
- HB, Electrolytes
- CBC
- PTT,PT

95- Which of the following regarding the Nsg diagnosis?


- Medical Pathology
- Treatment
- Actual problem
- Lab result

100- Health Education how to make wound care, the nurse knows that the
pt understands by:
- sfafes the sfeps ofsferf/e techniqtzes while rfea/ing with his wound.

101- to prevent lipo dystrophy with DM patient?


- Rofafe in/ecfion sites.
- deep injection
- use 25 gauge syringe.

102- Meningitis therapy (Nursing Care) includes:


- ventilate the room
- Allow frequent visitor.
- use low lighting system. (light sensitivity)

103- the purpose of giving “Anti D” for a pregnant woman?


- fo preyenf the RBCs destruction for fhe next 6a6y

104- a pregnant woman 2nd-3rd trimester, planned for C/S, the nsg priority is?
- Assess pain
- start IV fluids

105- Post normal vaginal Delivery, the pt developed vaginal bleeding, uterus is soft,
what is the most appropriate Nsg intervention?
- t/feros message to make fhe tzfertzs rigid and decrease 6/eecfing.

106- The most suitable diet for a woman with pre- exlampsia is?
- high protein, low sa/I r/iet

107- the reason of gum bleeding for a pregnant woman?


- high estrogen /eve/

108- 20 weeks pregnant woman, first fatal movement called?


- Quickening
109- when you let the patient suddenly down, the normal newborn's reflex is called?
(revise reflexes)
- Moro reflex
- Babiniski reflex
- rotating (sucking) reflex
- grasping

110- to prevent uterus laceration during


delivery... ‘- Episeotomy

111- Marker diagnostic investigation for Breast CA (Cancer) is?


- ERP test
- CD and T

112- the priority, pt with facial and chest burn is?


- maintain airways and breathing. (/aryngea/ edema)

113- Post ETT (Endotracheal Intubation), patient's breathing with gargling, this gargling
is evidence that the tube is located in:
- Bronchioles
- Trachea
- Carina
- Esophagous

114- the drug of choice for Supra ventricular tachycardia is


- D/C shock
- Atropine
- Adrenaline
- Adenosine

Adenosine, an ultra short acting AV nodal blocking agent, is indicated if vagal


maneuvers are not effective.[9] If successful, followup therapy with diltiazem, verapamil
or metoprolol may be indicated. Adenosine may be safely used during pregnancy.
[10]

SVT that does not involve the AV node may respond to other anti-arrhythmic drugs such
as sotalol or amiodarone.

115- the In charge nurse prepared a medication and asked the RN to give it to patient
in room 4, the appropriate RN intervention:
- refuse giving this medication ( who prepared will give, no deligation)
- give it, and sign instead of the in charge.
116- the first priority regarding medication administration ?
- chceck pts name
- check the expiry date
- check physician order
- check medication name

117- preparation for thoracentesis?


- give pre medication
- keep pt NPO for 8 hrs.
- keep the pt on upright position and mark the site.

118- the ideal way to remove the eye lenses?


- apply a pressure fo fhe eye/irfs fhen instruct fo clinch.

119- Documentation error (with 2 words) hoe the nurse fixes this error?
- use the corrector
- flat line over then sign

120- documentation- while the nurse document in a pts file, he discovered that he
was writing in the wrong pt, what is the appropriate action should the nurse do?
- make oblique line in the who/e page and sign.

Again, there are no answers for most of the HAAD exam questions above; do your
part.

You might also like